DC Dutta’s Clinics in Gynecology Hiralal Konar, DC Dutta
INDEX
Page numbers followed by f refer to figure, fc refer to flowchart, and t refer to table, respectively.
A
Abdominal incision 201
types of 202f
Abnormal uterine bleeding 29, 64, 94, 96, 154, 209
basic pathology of 98
causes of 96
classification of 94
differential diagnosis of 95
iatrogenic causes of 98
management of 98, 99
prevalence of 96
severity of 96
Abortion
aspiration 161t
follow-up care of 160
guidelines, WHO medical management of 158
incomplete 162
induced 159, 162
management of 158, 162
medical 157, 158, 160, 179
methods of 157
septic 162
surgical 157160
Acanthosis nigricans 75f, 78
Acne 78
Acquired immunodeficiency syndrome 50, 66
Actinomycin D 193
Activated protein C resistance 59
Add-back therapy 84
Adenocarcinoma 26, 26f
endometrial 29
primary peritoneal 38
serous 225
Adenomyomectomy 88
Adenomyosis 7, 88, 90, 95, 96, 98, 140
clinical features of 87
diagnosis of 87
management of 88, 89
risks factors for 87
surgical management of 88
treatment of 7
ultrasonographic diagnostic criteria of 87
uteri 87
Adnexal mass 100, 101f, 102
differential diagnosis for 100
management of 100, 102
Adoption 117
Adrenal disease 12, 115
Adrenal gland 35
androgen producing tumors of 78
Adrenogenital syndrome 13f, 44
characteristic features of 13
congenital 37
Alkali hematin 62
Alopecia 78, 190
Alpha-fetoprotein 102
Amenorrhea 3, 9, 98, 104, 113, 190
induction of 104
primary 12, 32, 36
secondary 14, 32, 34
American Society for Reproductive Medicine 130t
Amniotic sac 207f
Anal canal 54
Anastrozole 88
Androgen 39, 74, 85
binding proteins 123
exogenous 78
insensitivity syndrome 13
metabolism 48
role of 138
sources of 36
Androgenic-anabolic steroid 78
Anencephaly 51
Anesthesia, local 160
Aneuploidy screening 138
Angiogenesis 191
Anovulation 98, 117, 189
Anovulatory dysfunctional uterine bleeding 98
Anovulatory infertility, problems of 16
Antiangiogenesis agents 191
Antibiotics 215
broad-spectrum 147
prophylactic 214
regimens 216
Anticonvulsants 147
Antiemetic drugs 192
Antiestrogens 71
Anti-Müllerian hormone 89, 109, 116, 135, 137
levels of 135
physiological functions of 135
Antimuscarinic drugs 68
Antioxidant food supplement 124
Antiphospholipid antibody 58
syndrome 57, 58, 60, 149
Antiretroviral therapy 148
toxicities of 67
Antithrombin 59
Antral follicle count 115, 135
Aortic lymphadenectomy 26
APA syndrome, diagnosis of 60
Apgar score 268
Apoptosis 196
Aromatase inhibitors 71, 84, 88
Arterial cardiovascular disease 148
Arteriovenous malformation 97, 98
diagnosis of 97
Artificial urinary sphincter 69
Asherman's syndrome 44
Aspirin, low-dose 138
Assisted reproductive technology 16, 18, 75, 107, 119, 124, 126, 130t, 133, 134
cycle 89, 126, 127
principle steps of 17
different methods of 16
health risks of 133
procedure of 133
steps of 127t
treatment, adverse effects of 130t
Asthenospermia 123, 124
Asthma 92
Atresia 109, 111, 128
Autosomal chromosomal syndrome 52
Autosomal dominant inherited cancer 197
Azoospermia 124, 125
classification 124
testicular 124
treatment 124
B
Bariatric surgery 229, 230
place of 76
principle of 229
Bartholin's gland 40, 265
Basal serum
estradiol level 116
inhibin B 135
Beta-human chorionic gonadotropin 181
Bevacizumab 191
Biopsy 29, 95, 185
endometrial 62
multiple peritoneal 185
testicular 35
Bipolar electrosurgery 80
Bipolar radiofrequency endometrial ablation 210
Birth defects 67
Bladder injury 222
Blastocyst transfer 129
Bleeding 160
disorders 65
excessive 162
Blood 209
loss, measurements of 62
pressure 92
transfusion 133
Boari flap 224f
Boari method 223
Body
mass index 137
surface area 192
weight 137
Bone
marrow depression 190
mineral density 148
Bonney's gynecological surgery 267
Bonney's myomectomy 266
clamps 266
screw 266
Bonney's test 267
Botulinum toxin, injection of 69
Brachytherapy 194196
Brain metastasis 178f
Brandt-Andrews maneuvers 266
Braxton-Hicks contractions 266
Breast
atrophy of 78
cancer 148
disease, benign 92
examination, clinical 11
female 54
self-examination 11
Breastfeeding 146, 148, 245
Bromocriptine therapy 36, 37
Burch colposuspension 46
Buserelin 126
C
Call-Exner body 51, 52
Camper's fascia 201
Cancer
antigen 102
cervix 97
management of 23
familial 29, 30
gynecologic 197
testicular 155
Carbohydrate intolerance 189
Carboplatin 191, 193
Carcinoma
cervix
management of 23
pathogenesis of 22
serum tumor markers of 25
types of 21
embryonal 102
Cardiac failure, congestive 211
Cardiovascular disease 76, 156
Catastrophic antiphospholipid syndrome 57
Cavitation 218
Cell death 196
Cellular pleomorphism 174
Centchroman (Saheli) 147
Cerebral
canal 160, 207f
funneling of 207f
cancer 24, 183, 187, 195
brachytherapy for 195
elimination of 24
radiotherapy for 195
screening program 23, 41
cap 162
cerclage operation 206208
cytology screening 45
ectopy 96, 98
edema 211
fibroid 10, 10f, 70
incompetence 59, 206
insufficiency 206
intraepithelial neoplasia 32
pathology 96
preparation 160
Cervix 207f
carcinoma of 21, 21f, 2224, 24f, 25
microinvasive cancer of 43
squamous cell carcinoma of 21, 24
ultrasonography of 207f
Cesarean delivery 133, 134
Cetrorelix 126
Chemoradiation, concurrent 23, 26
Chemotherapeutic drugs 193
Chemotherapy 149, 186, 190, 192
prophylactic 49
response, evolution of 192
Cherney incision 202
Chest infection 243
Chlamydia trachomatis 17, 65, 245
Chocolate cyst 8f, 82, 100, 100f
Chorioamnionitis 243
Choriocarcinoma 35, 47, 49, 102, 177f, 178f, 181
Chorionic villus sampling 52, 53
Clean wound 216
Clear cell carcinoma 171
Clitoris, enlargement of 78
Cloaca 38
Clomiphene 118
citrate 138, 227
Coaptation 218
Cobra grasper 233f
Coffee bean nuclei 52
Colinidine 93
Colorectal cancer 29
Colporrhaphy, anterior 69
Colposcopy 51
Coma 211
Combined oral contraceptives 64, 70, 81, 84, 98, 126, 128, 135, 146, 163
pill 6, 45
Complete mole, karyotype pattern of 10f
Computed tomography scan 29, 65
Condom 162, 163
Conformal radiation technique 195
Contaminated wound 216
Contraception 104, 143, 145, 152, 211
advances in 145
emergency 104, 162
hormonal 147
long-term reversible methods of 153
methods of 149, 163
permanent method of male 148
post-abortion 162
Contraceptive
long-acting reversible 148150
methods 162
failure rates of 51
pills, oral 137, 138
Contrast-enhanced computed tomography 97
Controlled ovarian stimulation 81, 121, 126, 138
Copper-bearing intrauterine device 163
Cornual ectopic pregnancy 236f
Coronary artery disease 117
Corporeal fibroids 70
Corpus luteum 31
formation of 115
Corticosteroids 117
Couvelaire uterus 267
Craniopharyngioma 44
Cryopreservation, methods of 130
Cryosurgery 39
Cryptomenorrhea 12, 14, 15
Cushing's disease 78
Cushing's syndrome 78
Cut380 A 98, 145, 145f
Cyclooxygenase 99
Cyclophosphamide 186, 190
Cyst
aspiration 80
endometriotic 81
multiple small 73f
paraovarian 33, 101f
simple 101
Cystectomy 80
Cystic fibrosis 124
transmembrane regulator gene 124
Cystitis
hemorrhagic 190
interstitial 46, 83
D
Danazol 6, 88, 99
side effects of 6
Deep dyspareunia 5
causes of 37
Deep perineal pouch 39
Deep thermal injury 224
Dehydroepiandrosterone 117
Depot medroxyprogesterone acetate 70, 84, 163
Detrusor overactivity 46
Diabetes mellitus 92, 117, 189
gestational 76, 118, 133
juvenile 12
non-insulin-dependent 74
Diaphragm 162, 163
Diarrhea 160
Diathermy 153, 155
Dienogest 84
Dirty wound 216
Distal tubal intraepithelial carcinoma 225
Docetaxel 193
Dominant follicle
basically selection of 110
emergence of 109
selection of 115
Donor
insemination 125
oocyte 117, 131
Down's syndrome 52, 134
Doxorubicin 193
hydrochloride liposome 191
Drugs 193
adverse effects of 205
dose calculation 192
therapy 69
Dyschezia 83, 104
Dysfunctional uterine bleeding 105, 209
Dysgenetic gonads 14
Dysgerminoma 102
Dyslipidemia 74, 117, 189
Dysmenorrhea 83, 87, 104
intractable 7f
primary 49
Dyspareunia 83, 87, 104
causes of 8
severe deep 8
E
Ectopy 37
Electrical vacuum aspiration 161
Elevated maternal serum alpha fetoprotein level 51
Embryo 130f
cryopreservation of 130
culture 129
morphology 129
quality, morphological assessment of 129
transfer 117, 127, 130
number of 130t
procedure for 17, 130
Endocrine 109
abnormalities 5
factors 59, 61
Endodermal sinus tumor 102
Endometrial ablation 88, 99, 209
indications for 210
method 210
procedures 211
Endometrial cancer 20, 29, 105, 167, 183, 189
diagnosis of 20
evaluation of 21
management of 170, 172
surgical staging of 197
treatment of 167, 172, 173
Endometrial carcinoma 11, 19, 20, 26, 27, 27f, 31, 43, 96, 167f, 168, 170, 189
diagnosis of 20
high-risk factors for 26
histological type of 26
management of 168, 170
surgical management of 170
Endometrial destruction
commonly used first-generation methods of 210
second-generation methods of 210
techniques 210
Endometriomas 8082
laparoscopic cystectomy for 8
large 82
resection 85
Endometriosis 5, 7, 48, 80, 81, 83, 85, 104, 105, 120
complications of 5
deep infiltrating 83
diagnosis of 83
hysterectomy for 85
mild 8, 80
minimal 80
moderate to severe 81
severe 80
sign of 83
sites of 5
surgical procedures for 85
symptom of 83
treatment of 53
Endometritis 243
Endometrium 9f, 26, 97
antiproliferative effect on 104
carcinoma of 26, 167
complications of transcervical resection of 211
ectopic 83
evaluation of 97fc
laser ablation of 210
transcervical resection of 45, 63, 210
Epithelial ovarian
cancer 28
epidemiology for 28
tumor 53
Epithelium, types of 45
Erythropoietin, synthetic 192
Estradiol 110
Estrogen 91
dominant follicular microenvironment 111
secretion 38
therapy 91
benefits of 91t
route of 91
Ethamsylate 99
Ethinyl estradiol 146
Etoposide 191, 193
External beam radiation therapy 194
F
Facial hair
excess growth of 3f
male type of 75f
Fallopian tube 40, 50, 153f, 265
cancer 97
carcinoma of 36
Falope ring 153
Familial ovarian cancer 19, 101
Fascial interposition 155
Fatigue 191
Femoral artery catheterization 236f
Ferriman-Gallwey score 78
modified 78f
Fertility
awareness methods, use of 149
conserving surgery 185
preservation 175, 183
sparing surgery 25, 175
status 26
Fetal
chromosomal abnormalities 58
heart rate abnormalities, causes of 54
malformations 52
testis 37
trisomy, risks of 134
Fever 160
Fibroid 90, 94f, 96, 140
asymptomatic 70
location of 70
symptomatic 70
uterus 7, 62f, 70
management issues 70
Filshie clip 153, 153f
loaded applicator 153f
Fimbrioplasty, laparoscopic 18f
First generation ablation techniques 63
Fitz-Hugh-Curtis syndrome 34
Fluid over load 211
Folic acid, administration of 124
Folinic acid 190
Follicle-stimulating hormone 44, 98, 109, 110, 112, 123, 126, 227
Follicular androgens, role of 138
Follicular atresia 109
Follicular growth 109, 110f, 111, 128
monitoring of 17
phases of 109, 110f
process of 110
Folliculogenesis 109, 138
Frozen embryo transfer, endometrial preparation for 131
G
Gabapentin 93
Galactorrhea 115
Gamete intrafallopian transfer 127
Ganirelix 126
Gemcitabine 190
Genital organs
carcinoma of 220
development of 41, 49
female 39, 47
Genital tract, developmental defect of 12
Genital tuberculosis 31, 33, 37, 39, 41, 118
Genitalia
ambiguous 49
external 38
Germ cell 109
migration 109
tumor 50, 53
malignant 53
Gestational age
duration of 157
lower 133
Gestational trophoblastic disease 47, 149, 177, 180, 181
follow up of 180
management of 181, 182
treatment of 179
Gestational trophoblastic neoplasia 97, 177, 181
Gestrinone 85, 99
Glands, endometrial 87
Glomerular filtration rate 191
Glucocorticoid receptor 104
Glutaraldehyde cross-linked collagen 69
Glycine overload 211
Gonadotropin 109, 118, 126t
releasing hormone 44
agonist 84, 117, 127, 137, 184
analog 6, 16, 44, 64, 70, 81
antagonist 84, 126128, 138
therapy 123
stimulation 128, 137
therapy, indications of 16
Gonads, development of 49
Gonorrhea 245
Goserelin 126
Graafian follicle 265
Granulocyte colony-stimulating factors 192
Granulosa cell 31
Greater vestibular glands 265
Growth
factors 192
hormone 109
Gynecological cancers 190
radiotherapy in 194
Gynecological surgery, laparoscopic 223
Gynecology, endoscopy in 218
H
Hair
growth 78
rapid 78
removal 215
Harmonic ultrasonic energy 218
Headache 243
Heart 91
disease, ischemic 148
Heavy menstrual bleeding 62, 62f, 64, 209
operative treatments for 63
pharmacotherapy for 63
surgical management of 209
Hegar's dilator 266
Hematocolpos 14, 14f
Hematuria 104
Hemolysis 211
Hemorrhage 211
Hemorrhagic disorder 64
Hemostatic clips 218
Heparin, unfractionated 60
Highly active antiretroviral therapy 67
Hilus cell tumor 78
Hirsutism 78
ovarian causes of 35
Hobnail cells 52
Hormone replacement therapy 11, 46, 98
Hostile cervical mucus 124
Hot flushes 91
Human chorionic gonadotropin 177, 181, 184
Human growth hormone, role of 138
Human immunodeficiency virus 66
infection, congenital 66
Human leukocyte antigen 57
Human menopausal gonadotropin 126
Human papillomavirus infection, pathogenesis of 22f
Human progesterone receptor 104
Human seminal fluid 42
Hydatidiform mole 9, 9f, 177
Hydrocephaly 10
Hydrosalpinx 17, 17f, 18, 119
Hydrothermal balloon ablation 210
Hydroxylase deficiency 13f
Hydroxysteroid dehydrogenase deficiency 78
Hyperammonemia 211
Hyperandrogenemia 4, 74, 78, 117, 189
biochemical evaluation of 78
clinical features of 78
differential diagnosis of 78
gestational 79
Hyperhomocysteinemia 59
Hyperinsulinemia 4, 37, 74
Hyperlipidemia 92
Hyperplasia 87, 95
Hyperplasia
adrenal 35
congenital adrenal 13f, 45, 78
endometrial 11, 87
Hyperprolactinemia 115
Hypertension 92, 117
gestational 133
Hyperthyroidism 98, 115
Hypertrophy 87
Hypogonadism 44
hypergonadotropic 12, 115, 116
hypogonadotropic 12, 115, 116
Hyponatremia 211
Hypothalamic-pituitary
dysfunction 115, 117
failure 115
Hypothesis, hormonal 225
Hypothyroidism 98, 115
Hysterectomy 11, 19, 26, 64, 72, 85, 88, 140, 212, 213
abdominal 212
benefits of 19
bilateral oophorectomy during 19
indication of 64, 189
laparoscopic 212
operation of 34
radical 24, 24f, 168, 266, 267
risks of 19
robot-assisted 170, 213
route of 64, 212
total 34, 172
abdominal 29, 100, 168, 189
laparoscopic 170
types of 22
Hysterosalpingography 60f
Hysteroscopy 20, 26, 29, 60f, 63, 65, 95
advantages of 98
limitations of 98
I
Idiopathic thrombocytopenic purpura 96
Ifosfamide 190
Immature oocytes, collection of 118
Immunotherapy 60
Implants 147, 150
In vitro fertilization 82, 117, 119, 124, 127
cycle 17
embryo transfer, indications of 16
failure of 125
insemination 129
place of 81
surrogacy 140
In vitro maturation 183
Indocyanine green 197
Infections 60, 9698
control of 158
gynecological 245
prevention of 158
role of 121
sites of 242
types of 245
viral 42
Infertility 3, 16, 17, 32, 80, 83, 87, 107, 113, 124
causes of 5
evaluation 113
female 115
male 123, 124
management of 16, 113
peritoneal factors for 120
primary 16
tubal factors for 118, 119
unexplained 113, 121, 124, 125
Inflammatory bowel disease 83
Infracolic omentectomy 185
Inguinal canal 48
Injury, prevention of 221
Insemination 124
Insulin
growth factor 109
resistance 74
sensitizers, place of 4
Intensity-modulated radiotherapy 194, 196
Internal iliac artery 40
anterior division of 40
bilateral ligation of 53
International Federation of Gynecology and Obstetrics 178, 209
classifications 71, 71t
early stage disease 167
grading system 167
International Society for Minimally Invasive and Noninvasive Medicine 89
Intracavitary fibroids 71
Intracytoplasmic sperm injection 124, 127
fertilization rate of 125
procedure 125f
Intrauterine
contraceptive devices 51, 98, 145, 150, 153, 162, 163
fetal death 105
insemination 16, 81, 120, 124, 127
Intravaginal radiation 168
Invasive carcinoma 22f
cervix 39
management of 22
Invasive moles 181
Isosexual precocious puberty, causes of 38
J
Jadelle® implant 148f
Joel-Cohen incision 203
K
Kallmann's syndrome 44, 123
Karyotype 45
Keratin pearls 52
Klinefelter's syndrome 113, 124
Kroener technique 153
Krukenberg tumor 32, 266
L
Labia majora 48
Labia minora 48
Labor, induction of 134
Lactate dehydrogenase 102
Laparoscopic ovarian
diathermy 227
drilling 4, 33, 51
surgery 35, 51
complications of 228
Laparoscopic surgery 6, 40, 80, 176, 218, 220222
limitations of 234
Laparoscopic uterosacral nerve ablation 85
Laparoscopic-assisted
radical vaginal trachelectomy 26
supracervical hysterectomy 213
vaginal hysterectomy 170, 212
Laparoscopy 60f, 65, 85, 120
absolute contraindications of 39
gynecologic 220
indications of 85
Laparotomy 72, 174f, 175f, 176
standard procedure of 175
Laser surgery 80
Leiomyoma 70, 87, 94f, 95, 98, 105, 221f
management of 89
subclassification of 94
Leptin 116
Lesions, endometriotic 85
Leucovorin 190
Leukemia 96
Leukopenia 190
Leuprorelin 126
Levator ani 40
Levonorgestrel-releasing intrauterine device 62, 64, 70, 84, 88, 91, 99, 145f, 152
Leydig cell 123
tumor 78
Lippes loop 9f
Liver disease 98, 148
Low molecular weight heparin 60
Low-dose rate brachytherapy 194
Luteal phase defect 45, 122
treatment for 122
Luteinizing hormone 44, 98, 109, 110, 112, 123
Lymph node
approximate rate of 26
dissection 29
evaluation 172
involvement 172
para-aortic 29
place of 29
Lymph vascular space involvement 189
Lymphadenectomy 168, 197
bilateral extraperitoneal 267
para-aortic 168, 172, 197
place of 172
M
Madlener technique 153
Magnetic resonance imaging 27f, 29, 63, 65, 71, 83, 95, 97
Malaria 245
severe 245
uncomplicated 245
Malignancy 95
gynecologic 183, 185, 194
index, risk of 28
Marshall-Marchetti-Krantz procedure 69
Masculinization 78
Mass, presence of 100
Maternal sepsis 239, 241
Maternal serum alpha fetoprotein, low levels of 51
Matrix metalloproteinase 83
Mayer-Rokitansky-Küster-Hauser syndrome 12, 140
Maylard incision 202, 203
McCune-Albright syndrome 49
McDonald's operation 207
Medroxyprogesterone acetate 84, 99, 148
Meigs’ syndrome 53, 267
Menopause 11, 19, 34, 46, 91
Menorrhagia 7f, 87, 96f
Menotropin 126
Menstrual cycle 31, 109, 111
irregularities 78
normal 48, 49
Menstruation 115
abnormal 104
cessation of 14
normal 12
Mesonephric duct 265
Mesosalpinx 38
Metabolic syndrome 75, 117
Metallic-graded cervical dilators 266
Metformin 76, 117
dose schedule of 76
metabolic functions of 75
therapy 75
Methotrexate 186, 190, 193, 205
Metrorrhagia 87
Microinvasive carcinoma cervix, management of 22
Microsurgical epididymal sperm aspiration 125
Midurethral retropubic tape 69
Mifepristone 45, 70, 85, 88, 104, 105, 160
Migraine 92
Minimally invasive surgery 85, 167, 170
Mirena 98
Miscarriage 17, 58, 60f, 76, 87, 133
recurrent 57, 58, 60, 61, 76, 117, 140
Misoprostol administration
buccal routes of 160f
different routes of 160
sublingual routes of 160f
Missed pills 146
risks of 146
Mitotic catastrophe 196
Molar pregnancy 49, 149
partial 149
treatment of 179
Mucus debris 118
Mullerian duct system 50, 135
Multileaf collimator 194
Multiload 375 145, 145f
Mumps 113
Muscle transection incision 202f
Myolysis 72
Myoma 219f
enucleation of 218f
interstitial 120
submucosal 120
Myomectomy 63, 64, 211, 266
hysteroscopic 63, 212
laparoscopic 72, 218f, 219f
limitations of 90
Myometrium 26f, 27f
evaluation of 97fc
hypertrophic 98
N
Nafarelin 126
National Comprehensive Cancer Network 197
Natural killer cells 59
Nausea 160, 190, 191
Necrotizing fasciitis 216
Negative pressure wound therapy 215
Neisseria gonorrhoeae 245
Neoadjuvant chemotherapy 23, 29
place of 29, 195
Neonatal intensive care unit 133
Neoplasia, nongestational 182
Neural tube defects 59
Night sweat 91
Non-descent vaginal hysterectomy 212
Non-invasive perinatal testing 134
Nonpolyposis colorectal cancer, hereditary 197
Nonsteroidal anti-inflammatory drugs 64, 81, 84
Norethisterone 84, 99
acetate 84
enantate 163
No-scalpel vasectomy 155, 155f
Nuclear atypia 174
NuvaRing 150
O
Obesity 4, 74, 92, 96, 133, 189, 220, 229, 230
android 73
central 74
complications of 229
gynecoid 73
management of 229
neonatal effects of 229
Obstetrics and gynecology, history in 263, 265
Olaparib 191
Oligoasthenoteratozoospermia syndrome 41
Oligomenorrhea 3, 113
Oligoovulation 117
Oligospermia 124
Omentectomy 197
Omplanon® implant 147f
Oncology 19
gynecological 165
Oocyte
cryopreservation of 130, 183, 185
growth of 129
maturation of 129, 129f
primary 109
retrieval 129
Oophorectomy 19, 64
Oophoropexy 185
Open surgery 80
Operative gynecology 199
Oral biguanide 117
Oral progestin pills 84
Oral progestogen 64
Orchitis, history of 113
Ormeloxifene 147
Osmotic dilators 160
insertion of 160
Osteoporosis 11, 60, 92
postmenopausal 46
Ovarian
cancer 19, 2729, 38, 41, 183, 185, 186, 197, 225
low-grade serous 226, 226t
management of 29
staging of 185
carcinoma 225, 226
types of 225
cycle, physiology of 115
cyst 102, 121
complex 101
cystectomy 80
drilling 76
dysfunction 5
electrodiathermy, place of 76
failure, premature 32, 92, 115
fimbria 101f
follicular development 47
germ cell tumors 102
hyperstimulation syndrome 17, 89, 126, 133, 135, 136, 138
insufficiency 117
ligament 45
malignancy 28, 185
mass 175f
reserve 113, 135
assessment of 116
normal 136
prediction of 135
tests 116
response, prediction of 135
stimulation 128
stroma 73f
suppression 185
surgery 80
conservative 185
tissue cryopreservation 183, 185
transposition 184, 185, 187
tumor 28f, 36, 45, 53, 174, 174f, 175f
borderline 174
malignant 185
volume measurement 135
Ovary 50, 112
androgen producing tumors of 78
carcinoma of 27
chocolate cyst of 82, 100
development of 41
feminizing tumors of 51
follicular cysts of 31
germ cell tumor of 50
granulosa cell tumor of 34, 50
hormone producing tumors of 51
neoplasms of 43
sex cord stromal tumors of 36
virilizing tumors of 51
Ovulation 42, 49, 109, 111, 115
endocrine control of 109, 110
failure of 112fc
induction of 4, 16
inhibition of 104
physiology of 109
process of 111
suppression of 104
theory 225
Ovulatory cycle 109, 111
Ovulatory disorders 16, 115, 116
classification of 115
management of 116
Ovulatory dysfunction 95, 115
Oxaliplatin 186
Oxygen
dissociation curve 54
therapy 242
P
Paclitaxel 193
Paget's disease 37
Pain 160
causes of 83
lower abdominal 8, 14
management of 83, 158
relief, surgical management of 85
treatment of 85
Palmar plantar erythrodysesthesia 192
Papillary serous cyst adenocarcinoma 101
Para-aortic node 170, 171, 185
Paramedian incision 203
Partial mole 9, 10, 10f
triploid chromosomal pattern of 10f
Pediatric human immunodeficiency virus, stages of 66
Pelvic 168
adhesions 220
angiography 236f
endometriosis 5, 5f, 80, 87, 118, 220
diagnosis of 5
surgery for 6
symptoms of 5
therapy for 5
treatment of 6
endometriotic lesions, naked eye appearance of 5
examination 12, 14
floor muscle exercise 68
infection 45, 118, 160
inflammatory disease 83, 243
lymph nodes 170
lymphadenectomy 24, 197
magnetic resonance imaging 187
malignancy, staging of 31
organ prolapse 220
pain 5, 8, 11, 83, 84, 100, 104
pathology 80t, 220
ureter 39
Pelvis
arterial supply of 50
laparoscopic view of 119f
ultrasonography of 4f
Percutaneous epididymal sperm aspiration 125
Peritoneal fluid cytology 185
Peritoneal oocyte and sperm transfer 127
Peritoneum 202
Pfannenstiel incision 201
Phimotic ostium 18f
Photophobia 243
Pictorial blood
assessment chart 96f
loss chart 62
Pinard's maneuvers 266
Pipelle endometrial sampling, limitations of 98
Pituitary tumor 115
Placenta
accreta 33, 133
previa 133
Placental site trophoblastic tumor 177, 179, 181, 182
Plasma 80
Platelet
count 65
dysfunction of 99
Pneumoperitoneum 40
Polycystic ovarian disease 267
long-term consequences of 76
surgical management of 227
Polycystic ovarian syndrome 3, 14, 37, 43, 73, 75, 76, 78, 96, 111, 113115, 117, 126, 135, 189, 227
diagnosis of 32, 73
etiopathology of 73
management of 4
symptoms of 3
Polypoid mass, irregular 26
Polyps, endometrial 98
Pomeroy technique, modified 153
Pomeroy's method 33, 153, 153f
Positron emission tomography scan 29
Postcoital vaginal bleeding 21
Postmenopausal bleeding 9f, 11, 19, 26, 20f
causes of 8, 11, 19
Postoperative surgical site infections 214
Postpartum hemorrhage 133
Post-testicular azoospermia 124
Post-tubal sterilization syndrome 154
Postvasectomy considerations 155
Pouch of Douglas 265
Pre-eclampsia 133
Pregnancy 242
after endometrial ablation 211
complications 211
early diagnosis of 134
ectopic 47, 133, 204
loss 53
recurrent 57
luteal phase 154
medical termination of 162, 179
of unknown location 205
persistent ectopic 205
prevention of 104
rates 131
second trimester termination of 159
test 65
Premature luteinisation 128
Presacral neurectomy 85
Preterm labor 133
Pretesticular azoospermia 124
Primordial follicles 109
Progesterone 61
only pill 84
use of 70
receptors 104
activation of 104
supplementation 122
Progestins 84
only pill 98
replacement study 91
therapy 189
Progestogen 91, 99
only injectable contraceptive 150, 163
Prograsp forceps 233f
Prolactinomas 43
Prostate cancer 155
Psammoma bodies 52
Pseudohermaphroditism, male 44
Pseudomyxoma peritonei 53
Psoas hitch method 223
Pubertal ovary 186
Puberty 36, 109
precocious 36, 44
Pubic hair distribution, male pattern of 75f
Pudendal nerve 40, 53
supplies, sensory component of 40
Pyosalpinx 17
Q
qSOFA score 241
R
Radiation therapy 168, 195t, 196
Radiotherapy
advantages of 21
postoperative 194
preoperative 194
Raloxifene 93
Rapamycin inhibitors, mammalian target of 191
Rectum 54
Rectus
muscle 201
sheath 201
Reinke's crystal 52
Renal dysfunction 98
Reoxygenation 196
Reproductive endocrinology 135
Reproductive organs, male 123
Retinoblastoma 22
Retroperitoneal lymph node sampling 185
Robertsonian translocation 58
Robotic surgery 170, 232
advantages of 233
complications of 233
disadvantages of 233
overall advantages of 234
Robotic system, third generation 233
Robotic technology 232
Rollerball endometrial ablation 210
Rosiglitazone 117
Round ligament 40, 50
Rubber ring pessary 20f
Rubella
infection, maternal 52
syndrome, congenital 52
Rutledge's classification 22
S
Sacral nerve stimulation 69
Saline infusion sonography 20, 63, 96
contraindications of 96
disadvantages of 96
Salpingectomy 205f
Salpingitis isthmica nodosa 118
Salpingo-oophorectomy 225
bilateral 29, 172, 100, 168, 172, 189
Sarcoma botryoides 43
Sayana press 148
Scalpel method 155
Scanty pubic hair 13f
Scar endometriosis, treatment for 6
Scarpa's fascia 201
Schauta-Amreich operation 266
Schiller-Duval body 52
Sclerosis, multiple 92
Second-generation endometrial ablation methods 63, 210
Selective progesterone receptor modulators 70, 85, 104, 105
contraindications of 105
use of 104
Semen analysis 123
Semicircular fold of Douglas 265
Sentinel lymph node 173, 197
Sepsis 241, 243t
management of 241
organ site of 243t
pathway 241
recognition 241
risk factors for 242
screening tools 242
Septic shock 241
multi-disciplinary team management of 242
signs of 242
Sequential organ failure assessment score 241
Serous ovarian cancers, high-grade 225, 226, 226t
Sertoli cells 123
Serum anti-Müllerian hormone 116
Serum human chorionic gonadotropin, estimation of 204
Sex
development, disorder of 49
hormone binding globulin 44, 79
Sexual development, female 45
Sexual dysfunction, male 113
Sexually transmitted infections 113
Sheehan's syndrome 34
Shirodkar's operation 207
Shock 54
Sickle cell anemia 92
Signet ring cells 52
Sims’ position 265
Sims’ speculum 265
Sims’ wire 265
Sinus, urogenital 38, 41
Skin
bruising of 78
changes 78
closure 215
Langer lines of 201
preparation 214
thinning of 78
Sleep disturbances 91
Somatotropic axis support 110
Sonography
abdominal 101
transvaginal 3, 96, 97fc
Sonohysterography, saline infusion 65
Sperm
cryopreservation of 183
recovery, methods of 125
Spermatogenesis 47
Spermicide 163
Stein-Leventhal syndrome 36, 267
Stereotactic body radiation therapy 195
Sterilization 118
concurrent method of 154
female 152, 153f, 162
laparoscopic method of 153, 153f, 154
male 152
surgical 163
voluntary 155
Stress urinary incontinence 42, 46, 68
surgical procedures for 69
Struma ovarii 52
Subfertility, causes of 80, 80t
Submucous fibroids 71
Sunitinib 191
Superficial inguinal glands 40
Superovulation 81, 121
Suprapubic transverse incision 201
Surgery 120
contraindications of 167
gynecological laparoscopic 220, 221
nonhysterectomy 63
plan of 175
types of 71
Surgical site infection 214
classification of 215
management of 216
prevention of 216
risk factors for 214t
Surrogacy 140, 141
dilemmas of 140
ethical issues of 140
gestational 140
procedure 140
traditional 140
Syndactyly 10
T
Tamoxifen 71, 98
Taxanes 186
Tension-free vaginal tape 69
Teratospermia 124
Testicular feminization syndrome 13, 13f, 33
Testicular sperm
aspiration 125
extraction 125
Testicular tissue cryopreservation 183
Testis, function of 41
Testosterone, exogenous 124
Theca cell tumor 36
Thermal
balloon ablation 210
injury 223, 224
Thiazolidinediones 117
Thrombasthenia 99
Thrombocytopenia 60, 96, 99, 190
Thromboembolism, venous 92, 133
Thrombophilias 57, 59
acquired 57
diagnosis of 60
inherited 5759
Thrombophilic syndrome 117
Thyroid 12
disease 92
dysfunction 65, 96, 98, 115
function 47, 54
gland 266
hormones 54
stimulating hormone 65
Thyrotoxicosis 36
Tibolone 91
Tissue
injuries 193
inspection of 161
sampling methods 65
Topoisomerase inhibitors 190
Topotecan 191, 193
Toxic shock syndrome 47
Trachelectomy, radical 26, 187
Tranexamic acid 64, 99
Transabdominal procedure 207
Transcervical sterilization 148
Transforming growth factor-beta 109, 135
Transobturator tape 69
Transureteroureterostomy 223f
Transvaginal scan 205
Transverse incision
advantages of 203
disadvantages of 203
Trichomonas vaginalis 245
Trophoblasts, persistent 205
Tubal dysfunction 5
Tubal ectopic pregnancy 204
emergency laparotomy for 205f
medical management of 204
Tubal endometriosis 118
Tubal intraepithelial carcinoma 225
Tubal occlusion 152, 154
hysteroscopic method of 153
long-term risk of 154
methods of 153
timing of 154
Tubal patency tests 113, 119
Tubal polyps 118
Tubal reconstructive surgery, contraindications of 38
Tubal sterilization
failure of 154
procedure 35, 153, 154
reversal of 35, 119
Tubal surgery 118, 119
advantages of 119
disadvantages of 119
Tubectomy 163
Tubercular salpingitis 119f
Tuberculosis 12, 118
Tuberculous salpingitis 31, 118
Tubo-ovarian mass 97
Tuboperitoneal factors 113, 119f
Tumor 197
endometrial 27f
epithelioid trophoblastic 179
invades cervical stroma 168
low malignant potential 174
marker 28
Turner's syndrome 49, 267
characteristic features of 12
features of 13f
Twin pregnancy 178, 179f, 180
U
Uchida technique 153
Ulipristal acetate 104
Ultrasonography 4f, 9f, 65, 83, 130, 207f, 209
Unruptured tubal ectopic pregnancy 205f
expectant management for 205
laparoscopy view of 204f
Ureter 33, 223
accidental injury of 33
delayed ischemic necrosis of 223
reimplantation of 224f
thermal injury of 224
Ureteric injury 34, 220, 222, 223, 223f
identification of 223
management of 223, 224
sites of 222
types of 222
Ureteric obstruction 224
Ureteric repair, surgical methods of 224
Ureteroneocystostomy 224f
technique for 223f
Urethral competence, tests of 69
Urinary bladder 53
Urinary fistula, swab test for 46
Urinary incontinence 46, 68
management of 69
mixed 68
Urinary tract injury 220, 221
causes of 221
identification of 222
management of 222, 223
prevalence of 220
prevention of 220
types of 220
Urine
analysis 68
involuntary leakage of 68
Urodynamic stress incontinence 36
sling procedure for 34
Urogenital system, embryology of 33
Uterine
anomalies 60
arteriovenous malformation 96
causes of 97
artery 24f, 50
embolization 42, 63, 64, 71, 88, 90, 97, 235, 236, 236f
ligation 88
cavity 39, 97fc
evaluation 95
factors 14, 120
fibroids 236f
management of 71, 90
treatment of 42
fundus 205f
perforation 211
sarcoma 97
synechiae 120, 140
Uterus 94f, 95f
enlarged 96
evacuation of 161
leiomyoma of 105
multiple fibroids of 100f
septate 60f
V
Vagina 50
agenesis of 33
carcinoma of 47
complete absence of 12f
Vaginal agenesis 12
congenital 32
Vaginal bleeding 9
Vaginal cleansing 215
Vaginal cuff brachytherapy 171
Vaginal discharge, persistent offensive 21
Vaginal examination 83
Vaginal hysterectomy 170, 212
limitations of 170
Vaginal ring 163
Vaginal trachelectomy, principal steps of 187
Vaginal vault brachytherapy 195
Vaginitis 43
Vaginosis, bacterial 43, 245
Varicocele repair 123, 124
Varicose veins 92
Vas
deferens, congenital bilateral absence of 124
dissection of 155f
division of 155
occlusion, methods of 155
Vascular endothelial growth factor 104, 191
Vasectomy 36, 124, 148, 149, 152, 155, 162
complications of 155
failure of 155
methods of 155
over tubectomy, advantages of 148
prevalence rate of 149
risks of 155
Vertical incision
advantages of 201, 203
disadvantages of 203
limitations of 201
Vesicovaginal fistula repair 265
Vincristine 193
Virilization 78
Voice, change of 78
Vomiting 160, 190, 191
von Willebrand disease 96
Vulva, Paget's disease of 37
Vulvodynia 48
W
Weight
disorders 115
gain 78
loss 117
Wertheim's operation 226
Wolffian duct 265
Wound
classification of 216
closure 203
infection 243
sites of 216
Wurm's operation 207
X
X-ray, chest 243
Z
Zinc, administration of 124
Zygote intrafallopian transfer 127
×
Chapter Notes

Save Clear


1Gynecology (OSCE)
Section Outline
  • Ch. 1. Objective Structured Clinical Examination (OSCE) in Gynecology
    • A. General Gynecology
      • Case 1: Polycystic Ovarian Syndrome (PCOS)
      • Case 2: Endometriosis—A
      • Case 3: Endometriosis—B
      • Case 4: Pelvic Pain
      • Case 5: Hydatidiform Mole
      • Case 6: Cervical Fibroid
      • Case 7: Menopause and Hormone Replacement Therapy (HRT)
    • B. Adolescent Gynecology
      • Case 8: Primary Amenorrhea
    • C. Menstrual Abnormalities
      • Case 9: Secondary Amenorrhea
      • Case 10: Hematocolpos (Cryptomenorrhea)
    • D. Infertility
      • Case 11: Infertility—A
      • Case 12: Infertility—B
    • E. Oncology
      • Case 13: Hysterectomy and Oophorectomy
      • Case 14: Postmenopausal Bleeding
      • Case 15: Carcinoma Cervix—A
      • Case 16: Carcinoma Cervix—B
      • Case 17: Carcinoma Cervix—C
      • Case 18: Carcinoma Endometrium
      • Case 19: Carcinoma of the Ovary
      • Case 20: Inherited Cancers and the Management
  • Ch. 2. Self-assessment in Gynecology2

Objective Structured Clinical Examination (OSCE) in GynecologyChapter 1

A. GENERAL GYNECOLOGY
 
CASE 1: POLYCYSTIC OVARIAN SYNDROME (PCOS)
zoom view
Fig. 1.1: Excess growth of facial (upper lip, cheek, chin and male-pattern beard) hair.
Q.1 What is the likely diagnosis?
Ans. Polycystic ovarian syndrome
Q.2 What is the current diagnostic criteria for PCOS?
Ans. According to ASRM/ESHRE, 2018 diagnosis of PCOS is based upon the presence of any two of the following three criteria:
  1. Oligo and/or anovulation
  2. Hyperandrogenism (clinical and/or biochemical)
  3. Polycystic ovaries.
Other causes of hyperandrogenic conditions (adrenal) are to be excluded.
Q.3 What are the symptoms of PCOS?
Ans.
  • Obesity
  • Infertility
  • Menstrual abnormality: Oligomenorrhea, amenorrhea, abnormal uterine bleeding (AUB)
  • Hirsutism
  • Characteristic skin changes (acanthosis nigricans)
  • Acne
  • HAIR-AN syndrome (See Dutta's Textbook of Gynecology, 8th Edition, p. 385).
Q.4 What are the characteristic changes in the ovary of a woman with PCOS?
Ans. Ovaries are enlarged in volume. The capsule is thickened. On cut section multiple (≥ 2) follicular cysts measuring about 2–9 mm in diameter are seen peripherally (Fig. 1.2). There is stromal hyperthecosis. Transvaginal sonography (TVS) can demonstrate the enlarged ovarian volume with peripherally arranged cysts.
zoom view
Fig. 1.2: Polycystic ovary.
Q.5 What other investigations would be appropriate for her?
Ans.
  • Ultrasound scan (TVS) (Figs. 1.3A and B)
  • To detect polycystic ovarian changes
  • Investigations for other factors for infertility, e.g., tubal patency test and husband's semen analysis.
Q.6 What is the basic underlying pathology in this condition of PCOS?
Ans. Hyperandrogenic state and chronic anovulation.
Q.7 What are the biochemical abnormalities seen in a case of PCOS?
Ans.
  • Hyperandrogenemia [(↑) testosterone, DHEA, androstenedione)]
  • Hyperinsulinemia (insulin resistance)
  • Hypersecretion of luteinizing hormone (LH)
  • 4Hyperprolactinemia
  • (↑) Serum estrogen
  • (↓) Sex hormone-binding globulin (SHBG)
  • (↑) Lipids.
zoom view
Figs. 1.3A and B: (A) Ultrasonography of the pelvis showing polycystic ovary (right); (B) Ultrasonography of the same patient showing polycystic ovary (left).Courtesy: Dr (Mrs) S Ghosh, Professor BN Chakravorty, IRM, Kolkata, India.
Q.8 How can obesity and hyperinsulinemia cause hyperandrogenism?
Ans.
  • Obesity → increased insulin resistance and raised insulin level stimulate → ovary (theca cells) → androgens (↑) : Obesity → (↓) SHBG → (↑) androgens (free).
  • Hyperinsulinemia → ovarian theca cells → (↑) androgens: increased insulin → (↓) hepatic synthesis of SHBG → more (↑) free androgens.
Q.9 What are the long-term consequences in a woman suffering from PCOS?
Ans.
  • Risk of developing diabetes mellitus due to insulin resistance.
  • Risk of endometrial carcinoma due to unopposed action of estrogen.
  • Risk of developing hypertension and cardiovascular disease due to abnormal lipid profile (dyslipidemia).
Q.10 What is the aim of management of PCOS?
Ans. Aim of treatment is to individualize the patient for her presenting symptoms like infertility, obesity or menstrual abnormality. In obese patients, weight reduction is essential.
Q.11 How do you treat hyperandrogenemia?
Ans.
  • Weight reduction
  • Combined oral contraceptive pills
  • GnRH agonists
  • Cyproterone acetate
  • Spironolactone
  • Flutamide.
Q.12 What medical management will improve her fertility status?
Ans. Weight reduction and induction of ovulation. Adjuvant drugs may be needed (e.g., metformin, bromocriptine) depending upon the associated abnormalities.
Q.13 If the woman is desirous of pregnancy how can you help her?
Ans. Ovulation of induction is to be done as she suffers from chronic anovulation. However, other factors (male factor, tubal patency tests) should be normal.
Q.14 What drug is commonly used for induction of ovulation?
Ans.
  • Clomiphene citrate
  • However, clomiphene citrate is to be combined with other drugs when other biochemical abnormalities are associated.
    • Clomiphene citrate + metformin → where there is obesity and hyperinsulinemia.
    • Clomiphene citrate + bromocriptine → where there is associated hyperprolactinemia.
Q.15 What is the place of insulin sensitizers in the management of women with PCOS desiring for conception?
Ans. Women with PCOS with body mass index more than 25 are often found insulin resistant. Along with weight reduction, treatment with metformin (insulin sensitizer) is found to reduce hyperinsulinemia and hyperandrogenemia. Pioglitazone and rosiglitazone are also being used in cases, resistant to metformin. Metformin is given in a dose 500 mg thrice daily.
Q.16 What are the different surgical methods?
Ans. Laparoscopic ovarian drilling (LOD) or laser (CO2) vaporization of the cysts is usually done. LOD reduces systemic and intraovarian androgen levels. The woman ovulates spontaneously following LOD (See Ch 54).
 
5CASE 2: ENDOMETRIOSIS—A
zoom view
Fig. 1.4: Pelvic endometriosis.
Q.17 What is the likely diagnosis?
Ans. Pelvic endometriosis.
Q.18 What is endometriosis?
Ans. It is the presence of functioning endometrium (both glands and stroma) in sites other than the uterine mucosa.
Q.19 What are the common sites of endometriosis?
Ans.
  • Ovary
  • Uterosacral ligaments
  • Rectovaginal septum
  • Pouch of Douglas (POD)
  • Sigmoid colon
  • Abdominal scar.
Q.20 What are the other symptoms?
Ans. Abnormal menstruation like menorrhagia, dysmenorrhea (secondary), dyspareunia, pain during defecation, rectal bleeding. She may be asymptomatic.
Q.21 What is the naked eye appearance of pelvic endometriotic lesions?
Ans. Lesions may appear as:
  • Small black dots (powder burns)
  • Red flame shaped areas
  • White patches
  • Peritoneal windows
  • Subovarian adhesions
  • Puckering of peritoneum
  • Ovaries may be involved—chocolate cysts
  • White peritoneal areas.
Q.22 What are the important symptoms of pelvic endometriosis?
Ans.
  • Some patients remain asymptomatic (25%)
  • Dysmenorrhea (50%)
  • Abnormal menstruation—menorrhagia, polymenorrhea
  • Infertility (40–60%)
  • Dyspareunia
  • Pelvic pain
  • Abdominal pain
  • Bladder symptoms (dysuria)
  • Bowel symptoms (rectal bleeding)
  • Pain during defecation.
Q.23 How the diagnosis of pelvic endometriosis is confirmed?
Ans. It is commonly done by diagnostic laparoscopy.
Q.24 What is the place of serum CA-125 in the diagnosis of pelvic endometriosis?
Ans. Measurement of serum CA-125 levels is not a diagnostic tool because of its low sensitivity (< 50%). It may be used to predict the recurrence of endometriosis after therapy.
Q.25 What are the causes of infertility in a woman with pelvic endometriosis?
Ans. The possible cause of infertility are:
  • Ovarian dysfunction: Anovulation, defective folliculogenesis or luteal phase defect.
  • Tubal dysfunction: Pelvic adhesions, causing distorsion of normal tube ovary relationship.
  • Others: Abnormal peritoneal fluid, implantation failure, miscarriage.
Q.26 What are the endocrine abnormalities associated with endometriosis?
Ans.
  • Corpus luteum insufficiency
  • Raised prolactin level
  • Luteolysis due to increased PGF2α.
Q.27 What associated condition such a patient may present with?
Ans. Infertility.
Q.28 What are the complications of endometriosis?
Ans.
  • Leakage or rupture of chocolate cyst
  • Infection of chocolate cyst
  • Infertility
  • Obstructive features (ureteral obstruction).
Q.29 What are the different modalities of therapy for pelvic endometriosis?
Ans.
  • Expectant
  • Medical NSAIDs: Hormonal
  • Surgery
  • Conservative
  • 6Combined surgical and medical
  • Definitive
  • Assisted reproduction (See Ch 23).
Q.30 What are the factors that determine the type of treatment to a particular woman?
Ans. To an individual woman the determining factors are:
  • Age of the woman
  • Size and extent of lesion
  • Severity of symptoms
  • Desire for a child
  • Stage of the disease [American Fertility Society (AFS) scoring system—See Dutta's Textbook of Gynecology, 8th Edition, p. 257]
  • Results of previous therapy.
Q.31 What are the different hormones that can be used for the treatment of pelvic endometriosis?
Ans.
  • Combined oral contraceptives (COCs)
  • Progestogens
    • Oral:
      • Medroxyprogesterone acetate
      • Dydrogesterone
      • Dienogest
      • Parenteral (IM): Medroxyprogesterone
      • Intrauterine contraceptive device (IUCD): Levonorgestrel intrauterine system (LNG-IUS)
    • Danazol
    • Gestrinone
    • GnRH analogs.
Q.32 What are the important side effects of danazol and GnRH analogs?
Ans.
  • Danazol: It causes symptoms due to pseudomenopause and it is less tolerated.
  • GnRH analogs: It works by creating medical oophorectomy compared to danazol, GnRH analogs are well-tolerated.
Q.33 What are the indications of surgery for pelvic endometriosis?
Ans.
  • Symptomatic endometriosis which is not responsive to hormone therapy.
  • Severe endometriosis—to correct the distortion of pelvic anatomy or for improvement of symptoms.
  • Chocolate cyst ≥ 4 cm needs ovarian cystectomy and adhesiolysis.
Q.34 What are the different types of surgery that can be done?
Ans. Either by laparotomy or laparoscopy.
Q.35 What is meant by conservative surgery?
Ans. Preservation of the reproductive organs and restoration of their anatomy for enhancement of fertility (function).
Q.36 What is meant by expectant treatment?
Ans. Expectant treatment means doing nothing actively. The woman is kept under observation.
Q.37 What is the place of expectant treatment?
Ans. Role of any treatment for minimal to mild endometriosis is controversial. Cumulative pregnancy rate is similar following expectant treatment and that after conservative surgery.
Q.38 Who are the cases for expectant treatment?
Ans. Minimal endometriosis when observed in
  • Unmarried women
  • Young married woman who is desirous of a baby
  • Women approaching menopause.
Q.39 What type of laparoscopic surgery is commonly done?
Ans. Destruction of endometriotic implants over the peritoneal surface is commonly done. It is done by diathermy or by laser vaporization. Ovarian endometrioma (chocolate cyst) can be resected laparoscopically. Laparoscopic uterosacral nerve ablation (LUNA) is done when pain is very severe.
Division of adhesions (adhesiolysis) can also be done by laparoscopy.
Q.40 What are the treatment options for this woman?
Ans. Combined oral contraceptive pill for a period of 6–9 months would be most appropriate for her. Continuous therapy without the usual 7 days break would make her amenorrheic. This will help regression of the endometriotic deposits. This will also improve her symptoms. At the same time contraception is also provided.
Other options depending on the severity of endometriosis would be:
  • Medical therapy: Continuous progestogen, dienogest or GnRH analogs.
  • Surgical therapy: Laparoscopic laser or electrodiathermy to ablate the endometriotic implants.
Q.41 What would be the appropriate management when she desires to conceive?
Ans. Pregnancy itself provides remission to the problem. For this case, following the initial treatment, expectant management would be appropriate for next 6 months. About 60–70% women will become pregnant within a year provided there is no other factor for infertility.
Q.42 What is the optimum treatment for scar endometriosis?
Ans. Treatment is done by excision. Hormone therapy is ineffective.
7Q.43 What is adenomyosis?
Ans. Adenomyosis is a condition when there is ingrowth of endometrium (both the glands and stroma) directly within the myometrium (See Ch 12).
Q.44 How does a woman with adenomyosis present?
Ans. Usually the woman is parous, with age above 40 years.
Common symptoms are:
  • Menorrhagia
  • Dysmenorrhea.
Q.45 How can you differentiate a fibroid uterus from an adenomyosis?
Ans. See Ch 12.
Q.46 What should be the treatment of adenomyosis in a parous and elderly woman?
Ans.
  • Treatment of adenomyosis (Figs. 1.5A and B) is predominantly surgical.
  • Medical management (hormones) is often ineffective.
  • Conservative surgery includes partial resection of adenomyomata.
  • Total hysterectomy with or without bilateral salpingo-oophorectomy is the optimum treatment for a woman who is parous and aged.
  • Currently, LNG-IUD is being used and found to be effective in improving the symptoms of menorrhagia and pelvic pain.
zoom view
Figs. 1.5A and B: (A) Postoperative specimen of a uterus with tubes and ovaries. Mrs SE, a 46-year-old, parous lady, was admitted for menorrhagia and intractable dysmenorrhea. She did not respond to any medical therapy. She was investigated. She underwent hysterectomy and bilateral salpingo-oophorectomy. The uterus is seen uniformly enlarged; (B) The specimen of the same patient is cut opened to show myohyperplasia with hemorrhagic spots within the myometrium.
 
CASE 3: ENDOMETRIOSIS—B
Q.47 What more relevant questions you will ask her to make the provisional diagnosis?
Ans. Regarding the pain, e.g., character of pain, duration, exact relationship with the period, deep dyspareunia and fertility status.
Q.48 What clinical signs will help you to make the differential diagnosis?
Ans. Pelvic tenderness, nodularity in the pouch of Douglas, fixed retroversion of the uterus, adnexal mass, tenderness and bulky uterus. The differential diagnosis includes pelvic endometriosis, adenomyosis, chronic pelvic inflammatory disease.
Q.49 To confirm the diagnosis what single investigation would be most valuable?
Ans.
  • Diagnostic laparoscopy by double puncture procedure.
  • Laparoscopy revealed the diagnosis (Fig. 1.6).
Q.50 Is this pathology going to affect her future fertility?
Ans. There is an association of pelvic endometriosis and infertility. Amongst the infertile patients, 15% 8suffer from endometriosis. Whereas patients with endometriosis suffer from infertility in 40–60% cases. The possible explanations are—anovulation, luteal phase defect, luteinized unruptured follicle syndrome, pelvic adhesions, dyspareunia, increased macrophage activity, altered prostaglandin balance and altered immune response.
zoom view
Fig. 1.6: Chocolate cyst of the right ovary (arrows).
Q.51 Should a patient with mild endometriosis be treated?
Ans. There is no advantage of any therapy over expectant management in cases with minimal or mild endometriosis. The association of infertility and endometriosis is not absolute unless there is tubal obstruction or extensive pelvic adhesions.
Q.52 What would be the appropriate treatment for this patient?
Ans. The content of chocolate cysts can be drained and the cavity is to be lavaged with normal saline. The lining of the cyst wall is then separated from the ovarian tissue. This may be done either by laparoscopy or by laparotomy. The principle of surgery is similar to ovarian cystectomy. Bleeding points are electrocoagulated by bipolar diathermy. In a small size chocolate cyst, the contents are aspirated and the cavity is irrigated repeatedly. This may be done under TVS guidance or laparoscopically under anesthesia.
Q.53 What is the overall result of laparoscopic cystectomy for endometrioma?
Ans. It is effective in relieving pain in about 74% of cases of mild to moderate disease. Pregnancy rate is about 60% in cases with moderate disease. However, improved pregnancy rate is observed within the first 6 months of conservative surgery.
 
CASE 4: PELVIC PAIN
Q.54 What are the common causes of acute pelvic pain in gynecology?
Ans.
  • Acute pelvic inflammatory disease (PID)
  • Ruptured chocolate cyst
  • Twisted ovarian cyst
  • Ruptured corpus luteum cyst
  • Disturbed tubal ectopic pregnancy
  • Ovarian hyperstimulation syndrome.
Q.55 What are the common causes of chronic pelvic pain?
Ans.
  • Dysmenorrhea
  • Premenstrual tension syndrome
  • Pelvic endometriosis
  • Ovarian remnant syndrome—trapped or residual ovarian syndrome
  • Psychosomatic
  • Pelvic vericosities
  • Uterine fibroid
  • Adenomyosis
  • Ovarian cyst
  • Pelvic inflammatory disease (PID) (chronic)
  • Retroversion or prolapse of the uterus
Q.56 What are the causes of dyspareunia?
Ans. Superficial
  • Narrow introitus
  • Tough hymen
  • Tender perineal scar
  • Vulval infection
  • Vulvar vestibulitis syndrome
  • Vaginal—septum, infection, agenesis.
Deep
  • Pelvic endometriosis
  • Chronic PID
  • Prolapsed ovary in the polycystic ovarian disease (POD).
Q.57 Mention some of the important causes of postmenopausal bleeding.
Ans.
  • Senile endometritis
  • Senile vaginitis
  • Dysfunctional uterine bleeding
  • Decubitus ulcer
  • Genital malignancy:
    • Carcinoma of the cervix, endometrium, vagina, vulva, fallopian tube
    • Uterine sarcoma.
  • Retained foreign body—pessary, IUCD (Fig. 1.7)
  • Urethral caruncle
  • Withdrawal bleeding following estrogen intake.9
zoom view
Fig. 1.7: Lippes loop. This has been removed from a 65-year-old lady hysteroscopically. She presented with postmenopausal bleeding. She had this at her age of 27 years. It was embeded within the endometrium and became brittle (arrows).
 
CASE 5: HYDATIDIFORM MOLE
Q.58 What is the provisional diagnosis?
Ans. Hydatidiform mole.
Q.59 What investigations should be done for her?
Ans. Ultrasonography of the uterus (Fig. 1.8) and urine or serum for β-hCG. Once the diagnosis is confirmed, other investigations are complete hemogram, ABO and Rh-grouping, LFT and X-ray chest and thyroid profile.
zoom view
Fig. 1.8: Ultrasonography—snowstorm appearance in hydatidiform mole.Courtesy: Dr (Mrs) S Ghosh, Professor BN Chakravorty, IRM Kolkata, India.
Q.60 What is the initial management for this problem?
Ans.
  • Correction of anemia with blood transfusion
  • To prevent infection
  • Suction and evacuation of the uterus with or without oxytocin drip should be done. Tissue should be sent for histological examination.
Q.61 What are the common complications of this problem?
Ans.
  • Hemorrhage
  • Infection
  • Pre-eclampsia
  • Perforation of the uterus
  • Respiratory distress due to pulmonary embolization of trophoblastic cells
  • Coagulation failure
  • Development of choriocarcinoma (2–10% cases).
Q.62 How should this patient be followed up and for how long?
Ans. Serial urinary β-hCG estimation weekly till negative. Usually, it becomes negative by 4–6 weeks time. Once negative, she is followed up at every month with serum hCG report for 6 months to 1 year.
Q.63 What precautionary measures she should be advised while on follow-up?
Ans. She should avoid pregnancy for at least 6 months to 1 year. For contraception, she can use barrier methods. Combined oral pills (low dose) may be used following normalization of β-hCG.
Q.64 What are the reasons that the patient needs to be followed up following initial treatment?
Ans.
  • Detection of cases with persistent trophoblastic disease.
  • Early detection of choriocarcinoma.
Q.65 What is her prospect of future pregnancy and chance of recurrence of the problem?
Ans. Prospect of future successful pregnancy is high, provided she is followed up. The risk of recurrence is less than 5%.
Q.66 What is the karyotype pattern of a complete hydatidiform mole and that of a partial mole?
Ans. In complete moles—karyotype pattern in majority is normal—46XX (90%). There is fertilization of ‘an empty ovum’ by a single sperm carrying 23 chromosomes. The usual 46XX chromosome pattern is the result of doubling of the paternal set of chromosomes (Fig. 1.9).
In about 10% cases, an empty ovum is fertilized by two sperm (dispermy), one carrying X and the other carrying Y chromosome. The chromosomal pattern is 46XY. All the chromosomes are derived paternally.10
zoom view
Fig. 1.9: Karyotype pattern of a complete mole—genome is entirely paternal in origin.
Partial moles (Fig. 1.10) consist of both the placenta and the fetus. Partial moles usually (90%) have triploid karyotype (Fig. 1.11). A normal ovum is fertilized by double sperm (dispermy), resulting in 69 chromosomes with sex chromosome configuration of 69XXX, 69XXY and 69XYY. The extra haploid set of chromosomes usually is derived from the father. The fetus is usually triploid, dies in the first trimester or is growth retarded with multiple malformations (syndactyly, hydrocephaly).
zoom view
Fig. 1.10: Partial mole with a stillborn baby.Courtesy: Dr S Mitra, Professor, Department of G and O, KPC Medical College, Kolkata.
zoom view
Fig. 1.11: Triploid chromosomal pattern of a partial mole. Genome is both paternal and maternal in origin.
 
CASE 6: CERVICAL FIBROID
Q.67 What is the diagnosis?
Ans. Posterior cervical fibroid.
Q.68 With what signs and symptoms she may have presented?
Ans. Vaginal discharge, pressure symptoms:
  • Bowel: Constipation, incomplete evacuation.
  • Urinary: Frequency, retention.
  • Lateral pelvic wall pressure to cause leg pain or edema.
zoom view
Fig. 1.12: Huge cervical fibroid.
Q.69 What surgery had been done for her?
Ans. Total abdominal hysterectomy with bilateral salpingo-oophorectomy.
Q.70 What are the dangers of this operation?
Ans. Hemorrhage, injury to bladder, rectum and the ureter.
Q.71 How the dangers could be minimized?
Ans.
  • Delineation of the course as well as the proximity of the ureters to the fibroid could be done preoperatively by intravenous urography.
  • Ureteric catheterization may be done preoperatively as a precautionary measure.
  • Enucleation of the myoma first followed by hysterectomy.
  • Preoperative GnRH analog therapy for 3 months may facilitate surgery.
 
11CASE 7: MENOPAUSE AND HORMONE REPLACEMENT THERAPY (HRT)
Q.72 What are the common causes of postmenopausal bleeding?
Ans. Senile vaginitis, senile endometritis, cervical carcinoma, endometrial carcinoma.
Q.73 What investigations should you organize for her?
Ans. Ultrasonography to assess endometrial thickness and/or any of the biopsy methods as:
  • Pipelle endometrial biopsy
  • Hysteroscopy and endometrial biopsy
  • Fractional curettage and endometrial biopsy.
Q.74 What abnormality will guide her to go for hysterectomy?
Ans. Significant endometrial pathology and risks of endometrial carcinoma are as follows:
Type of endometrial hyperplasia and the risk of endometrial carcinoma.
Typical
  • Simple (cystic without atypia): 1%
  • Complex (adenomatous without atypia): 3%.
Atypical
  • Simple (cystic with atypia): 8%
  • Complex (adenomatous with atypia): 29%
    The risk of malignancy with atypical hyperplasia is high. This may necessitate hysterectomy.
She is aware of menopause and osteoporosis. She wants to know the following.
Q.75 Has she got any of the risk factors for osteoporosis?
Ans. Risk factors are:
  • Family history
  • Early menopause
  • Low body weight
  • Excess caffeine intake
  • Smoking
  • Sedentary habit or use of some drugs (corticosteroids).
Q.76 What preventive measures can she take to prevent osteoporosis?
Ans. Exercise, adequate dietary intake of calcium, vitamin D and nonhormonal treatment (biphosphonate) or HRT.
Q.77 What HRT would be appropriate for her?
Ans. If the uterus is intact, combined estrogen and progestin cyclically. Progestin is added for last 12–14 days of each month or she can have low dose estrogen and progestin combined and continuous. Otherwise, following hysterectomy she should take estrogen only. There is reduction in the risk of osteoporosis, coronary heart disease and colon cancer.
Q.78 What other benefits can she have with HRT?
Ans.
  • Improvement of vasomotor symptoms (hot flushes, night sweats)
  • Prevention of atrophic changes in the skin, genital and urinary tract epithelium
  • Protection against cardiovascular disease
  • Reduction in the problems of anxiety, insomnia, irritability and depression.
Q.79 Is there any risk for continuing HRT?
Ans. In a well-selected case, risks are generally less compared to the benefits. In most of the studies (WHO 2003, Million Women Study 2003), there is no increased risk of breast cancer when estrogen is only used (in a hysterectomized woman). Currently low dose estrogen (conjugated equine estrogen 0.3 mg or ethinyl estradiol 1 mg) is recommended. However, the woman needs to be counseled.
But regular breast self-examination (BSE) monthly, clinical breast examination (CBE) yearly and mammography yearly (ACOG-2000) should be carried out as a part of breast screening.
There is an increased risk of deep vein thrombosis (DVT) from 10 to 30 per 10,000 users.
Q.80 What are the contraindications of HRT?
Ans. Contraindications are:
  • Undiagnosed genital tract bleeding
  • Estrogen-dependent cancer in the body
  • Active liver disease
  • History of venous thromboembolism
  • Gallbladder disease.
Q.81 What are different routes through which HRT can be administered?
Ans. HRT can be administered orally, through the skin as patches, gel, subcutaneous implant or nasal spray. It can also be administered by vaginal route as creams or pessaries. Use of oral route and the first pass liver metabolism has its beneficial effects on the lipoproteins.
Q.82 How long can HRT be taken?
Ans. The optimal duration of use for HRT is currently debatable. Small dose and short-term use for a period of 3–5 years has been recommended for bone protection.
12B. ADOLESCENT GYNECOLOGY
 
CASE 8: PRIMARY AMENORRHEA
zoom view
Fig. 1.13: Complete absence of vagina.
Q.83 What is most likely the diagnosis?
Ans. A case of vaginal agenesis.
Q.84 How do you define primary amenorrhea?
Ans. A young girl who has not menstruated by 16 years of age is defined as primary amenorrhea.
Q.85 When do you start investigations for primary amenorrhea?
Ans.
  • No menstruation by the age of 16 years, when other secondary sex characters are normal.
  • No menstruation by the age of 14 years when there is absence of growth and/or development of secondary sex characters.
Q.86 What are the factors essential for the onset and continuation of normal menstruation?
Ans.
  • Normal female chromosomal pattern (46XX).
  • Coordinated function of the hypothalamopituitary ovarian axis.
  • Anatomical presence and patency of the outflow tract.
  • Responsive endometrium.
  • Active support of thyroid and adrenal gland.
Q.87 What is cryptomenorrhea?
Ans. It is a condition where the menstrual blood fails to come out from the genital tract due to an obstruction present in the passage.
Q.88 What are the common causes of cryptomenorrhea?
Ans.
  • Congenital
    • Imperforate hymen
    • Transverse vaginal septum.
  • Acquired
    • Cervical stenosis following amputation
    • Vaginal stricture following traumatic (instrumental delivery).
Q.89 What are the common causes of primary amenorrhea?
Ans.
  • Hypogonadotrophic hypogonadism
    • Delayed puberty
    • Hypothalamic and pituitary dysfunction (stress, weight loss, anorexia nervosa)
    • Kallmann syndrome
    • CNS tumors—craniopharyngioma.
  • Hypergonadotrophic hypogonadism
    • Primary ovarian failure
    • Galactosemia.
  • Abnormal chromosomal pattern
    • Turner's syndrome (45X)
    • Various mosaic states (45X/46XX)
    • Testicular feminization (46XY).
  • Developmental defect of genital tract
    • Müllerian agenesis (MRKH syndrome)
    • Imperforate hymen
    • Transverse vaginal septum
    • Vaginal agenesis (Fig. 1.13).
  • Thyroid and adrenal disorders
  • Metabolic disorders (juvenile diabetes)
  • Systemic illness (tuberculosis)
  • Others: Uterine synechiae, unresponsive endometrium (receptor defect).
Q.90 What are the characteristic features of Mayer-Rokitansky-Kuster-Hauser syndrome?
Ans.
  • Stature—average
  • Breasts—normal
  • Sexual hair—normal
  • External genitalia—normal
  • Internal genitalia:
    • Vagina—absent
    • Uterus—absent/rudimentary
    • Ovaries—normal
    • Karyotype—normal (46XX)
    • IVP—Urinary tract abnormalities (30%)
  • Primary amenorrhea.
Q.91 What are the characteristic features of Turner's syndrome?
Ans.
  • Stature—short.
  • Secondary sex character—poorly developed
  • 13Webbing of the neck
  • Shield chest
  • Wide apart nipples
  • Cubitus valgus
  • Coarctation of aorta
  • ‘Streak’ gonads
  • Serum gonadotropins—high
  • Karyotype: 45XO or 45XO/46XX (Fig. 1.14)
  • Primary amenorrhea.
zoom view
Fig. 1.14: A 19-year-old girl with features of Turner's syndrome. Karyotype: 45XO.
Q.92 What are the characteristic features of androgen insensitivity syndrome [testicular feminization syndrome (Fig. 1.15)]?
Ans.
  • Stature—average
  • Scanty pubic and axillary hair
  • Breast development—normal
  • Vagina—short and blind
  • Uterus and tubes—absent
  • Gonads—at labia or intra-abdominal or at inguinal region
  • Gonadal biopsy—testicular tissue
  • Serum testosterone—equal to normal males
  • Karyotype: 46XY
  • Primary amenorrhea.
Q.93 What are the characteristic features of adrenogenital syndrome? (Fig. 1.16)
Ans.
  • Stature: Average
  • Phenotypically: Normal female
  • Labial fusion
  • Enlargement of clitoris
  • Uterus: Normal
  • Ovaries: Normal
  • Vagina: Normal
  • Karyotype: 46XX
  • Serum 17-OHP: Elevated
  • Urinary pregnanetriol: Elevated
  • Primary amenorrhea.
zoom view
Fig. 1.15: Testicular feminization syndrome showing short and blind vagina, scanty pubic hair, clitoromegaly and labial gonads (arrows).
zoom view
Fig. 1.16: A 14-month-old girl with adrenogenital syndrome showing labial fusion and enlarged clitoris. Congenital adrenal hyperplasia is commonly due to 21-hydroxylase deficiency.
14C. MENSTRUAL ABNORMALITIES
 
CASE 9: SECONDARY AMENORRHEA
Q.94 What is her present problem?
Ans. Secondary amenorrhea.
Q.95 How do you define secondary amenorrhea?
Ans. It is the absence of menstruation for 6 months or more in a woman who has menstruated normally in the past.
Q.96 What are the common ‘uterine factors’ that may cause secondary amenorrhea?
Ans.
  • Tubercular endometritis
  • Synechiae
  • Surgical removal (hysterectomy)
  • Transcervical resection of endometrium (TCRE).
Q.97 What are the ‘ovarian factors’ that may cause secondary amenorrhea?
Ans.
  • Polycystic ovarian syndrome
  • Premature ovarian failure
  • Surgical removal
  • Radiation.
Q.98 Dysgenetic gonads are the common ‘hypothalamic factors’ that may cause secondary amenorrhea?
Ans.
  • Stress
  • Anorexia nervosa
  • Strenuous exercise
  • Trauma
  • Infection (tuberculosis)
  • Psychogenic drugs (phenothiazine)
  • Tumors (craniopharyngioma, meningioma).
Q.99 What are the common causes of secondary amenorrhea?
Ans.
  • Stress
  • Polycystic ovary syndrome (PCOS)
  • Premature ovarian failure
  • Synechiae (Asherman's syndrome)
  • Drugs (phenothiazine)
  • Postpill amenorrhea.
Q.100 What is PCOS?
Ans. See Ch 8.
 
CASE 10: HEMATOCOLPOS (CRYPTOMENORRHEA)
Q.101 What is the diagnosis?
Ans. Imperforate hymen with hematocolpos.
Q.102 What other symptoms she might have?
Ans. Urinary retention, dysuria, lower abdominal heaviness and cyclical lower abdominal pain.
Q.103 What investigation you may do for her?
Ans. Ultrasonography of the lower abdomen to ascertain the extent of pathology, e.g., hematocolpos, hematometra, hematosalpinx.
Q.104 What would be the appropriate treatment for her?
Ans. Incision (cruciate) and drainage under general anesthesia, antibiotic coverage.
Q.105 What is the prospect of her future reproductive career?
Ans. Normal.
zoom view
Fig. 1.17: Hematocolpos showing the hymen tense, bluish and bulged.
15Q.106 What is the basic underlying abnormality of this condition?
Ans. It is due to failure of disintegration of the central cells of the Müllerian eminence that projects into the urogenital sinus. Depending upon the amount of blood so collected, it may present with hematocolpos (blood in the vagina), hematometra (blood in the uterus) or hematosalpinx (blood within the fallopian tubes).
Q.107 What is crypyomenorrhea?
Ans. There is periodic shedding of the endometrium and bleeding but the menstrual blood fails to come out from the genital tract due to obstruction in the passage.
Q.108 What are the common causes?
Ans.
  • Congenital:
    • Imperforate hymen
    • Transverse vaginal septum
    • Vaginal atresia in the upper part
  • Acquired:
    • Stenosis of the cervix following amputation, conization
    • Secondary vaginal atresia following difficult vaginal delivery.
16D. INFERTILITY
 
CASE 11: INFERTILITY—A
Q.109 According to WHO, how the ovulatory disorders are classified?
Ans. Group I: Hypothalamic-pituitary failure (hypogonadotrophic hypogonadism).
Group II: Hypothalamic pituitary dysfunction (normogonadotrophic normogonadism).
Group III: Ovarian failure (hypergonadotropic hypogonadism).
Q.110 How the problems of anovulatory infertility is treated?
Ans. Induction of ovulation is done. Different drugs are used depending upon the response. Clomiphene citrate is the commonly used drug. It is usually prescribed 50 mg twice daily from D-3 to D-7 (5 days) of the cycle. Ovulation usually occurs 5–7 days after the last day of therapy.
Q.111 What are the indications of gonadotrophin therapy for induction of ovulation?
Ans. Women with infertility due to:
  • Hypogonadotrophic hypogonadism (WHO group I)
  • Women who have either failed or are resistant to clomiphene citrate (WHO group II)
  • Women with unexplained infertility.
Q.112 When are GnRH analogs used for induction of ovulation?
Ans.
  • Patients who are refractory to gonadotropins.
  • Patients with elevated LH.
  • Patients with premature ovulation due to premature LH surge.
  • Patients with premature follicular luteinization.
Q.113 How do you manage the woman with WHO groups I, II or III ovulatory disorders?
Ans. WHO group I
  • Gonadotropins: Follicle-stimulating hormone (FSH), LH.
  • GnRH (pulsatile).
WHO group II
  • Clomiphene citrate
  • FSH
  • For premature LH surges—GnRH agonist/antagonist
  • For insulin resistance—insulin sensitizer
  • Adrenal androgens—glucocorticoids.
WHO group III
  • Cyclic hormone replacement therapy
  • Gonadotropins—high dose
  • Chance of spontaneous pregnancy—occasional
  • Assisted reproductive technology (ART) (See Ch 23)
  • Oocyte donation.
Q.114 What are the different types of surgery done for the management of infertility?
Ans. Surgery may be needed both for female as well as male factors for infertility.
For female
  • Laparoscopic ovarian diathermy (drilling) (LOD).
  • Wedge resection of ovary in cases with PCOS (not commonly done) (See Ch 54).
  • Surgery for pituitary prolactinomas.
  • Salpingo-ovariolysis—laparoscopically.
  • Proximal tubal block: Cannulation and balloon tuboplasty: This is done under hysteroscopic guidance.
  • Midtubal block: Recanalization procedure (reversal of tubal ligation).
  • Distal tubal block:
    • Fimbrioplasty
    • Neosalpingostomy
  • Tubal reconstruction (tuboplasty) following sterilization operation.
Q.115 What is assisted reproductive technology (ART)? What are the different methods of ART?
Ans. ART encompasses all procedures that involve manipulation of gametes and embryos for the treatment of infertility. For the rest (See Ch 23).
Q.116 What are the indications of in vitro fertilization-embryo transfer (IVF-ET)?
Ans.
  • Tubal disease
  • Unexplained infertility
  • Endometriosis
  • Male factor infertility
  • Cervical hostility
  • Failed ovulation induction.
Q.117 What are the indications of intrauterine insemination (IUI)?
Ans.
  • Cervical stenosis
  • Immunological factors for infertility (male and female)
  • Oligospermia or asthenospermia
  • Unexplained infertility.
17Q.118 What are the principle steps of an ART cycle?
Ans.
  • Down regulation using GnRH analog
  • Controlled ovarian stimulation (COS)
  • Monitoring of follicular growth
  • Oocyte retrieval
  • Fertilization in vitro [IVF, intracytoplasmic sperm injection (ICSI)]
  • Transfer of gametes or embryos
  • Luteal support with progesterone.
Q.119 How is the monitoring of follicular growth done?
Ans. It is done by:
  • Sonographic (TVS) measurement of follicular diameter
  • Serum estradiol level 3 or more follicles > 18 mm in diameter and serum E2 levels: 100–150 pg/mL/follicle is optimum. Injection hCG 5000–10,000 IU is given IM. Oocytes are retrieved 36 hours after hCG administration.
Q.120 What is the procedure for embryo transfer?
Ans. Transfer is done at 4–8 cell stage (48–72 hours later) transcervically. Usually two embryos are transferred.
Q.121 What is ovarian hyperstimulation syndrome (OHSS)?
Ans. It is characterized by multiple follicular development and ovarian enlargement following hCG stimulation. It may be a serious complication.
It is clinically manifested by bilateral ovarian enlargement, abdominal pain, nausea, vomiting, ascites, hypotension, hemoconcentration, oliguria, disseminated intravascular coagulation (DIC), thrombosis and sometimes adult respiratory distress syndrome.
Q.122 How can OHSS be prevented and managed?
Ans. See Dutta's Textbook of Gynecology, 8th Edition, p. 444.
Q.123 How endometrial receptivity could be improved in an IVF cycle?
Ans. Exact understanding of endometrial receptivity is lacking. Importance of many adhesion molecules (integrins) have been considered. However supplemental progesterone therapy is started after oocyte retrieval in all IVF programs as a routine. This is mainly because GnRH analog use and oocyte aspiration impair the secretion of endogenous progesterone from the ovary (corpus luteum).
Q.124 How is egg retrieval done in an IVF cycle?
Ans. Ultrasound-guided needle aspiration of oocyte through the vagina is done. This is done under IV sedation. The complications may be injury to bowel, bladder or infection.
Q.125 How can a man without any sperm to ejaculate, father children?
Ans. ICSI is the optimum procedure for this man, if he can produce sperm on testicular biopsy. ICSI is also helpful to men with congenital absence of vas deferens.
Q.126 Should a woman with hydrosalpinx be considered for IVF-ET?
Ans. IVF is considered to bypass the function of the blocked tubes. But presence of hydrosalpinges causes implantation failure with reduced pregnancy rate in IVF cycles. So, it is recommended to remove the tubes (prophylactic salpingectomy) prior to IVF in these women. The tubes may be clipped laparoscopically to get the same benefits.
 
CASE 12: INFERTILITY—B
zoom view
Fig. 1.18: Hysterosalpingogram showing bilateral hydrosalpinx without any spillage of dye.
Q.127 What is this investigation?
Ans. Hysterosalpingography.
Q.128 What is the pathology revealed by the procedure?
Ans. Bilateral hydrosalpinx without any spillage of dye.
Q.129 What is the basic pathology for this abnormality?
Ans. In majority, it is the sequelae of ascending infection from the lower genital tract. Chlamydia trachomatis (85%), N. gonorrhoeae and other pyogenic infections (E. coli) are responsible. Mixed infections (aerobic and anaerobic) are more common. Infection and inflammation cause destruction of tubal epithelium and pus formation known as pyosalpinx. There is 18peritubal adhesion formation and agglutination of the fimbriae. Gradually when the inflammation subsides, the pus settles down and the tube is filled with a clear fluid called hydrosalpinx. The tube is dilated, retort-shaped with closed abdominal ostium.
zoom view
Figs. 1.19A and B: (A) Laparoscopic fimbrioplasty by introducing the closed forceps within the closed ostium; (B) The phimotic ostium is opened up and the fimbrial folds are released by opening up the forceps blades.
Q.130 Is there any other method of evaluation?
Ans. Diagnostic laparoscopy and chromopertubation under general anesthesia. It may reveal tubal edema, hyperemia, dilated tubes (hydrosalpinx), fimbrial adhesion and/or peritubal adhesions.
Q.131 What are the treatment options that may resolve her problem of infertility?
Ans. Tubal reconstructive surgery (tuboplasty) may be attempted. This may be in the form of releasing the peritubal adhesions (adhesiolysis) and/or opening up the fimbrial end (fimbrioplasty or salpingostomy). As the tubal epithelium is often destroyed, the results of such operations are not always successful. Pregnancy rate following laparoscopic fimbrioplasty is 30–35%. Ectopic pregnancy rate is 5–10% (Figs. 1.19A and B).
Q.132 What other options are left to her?
Ans. Assisted reproduction—IVF-ET.
Q.133 What are the different methods of assisted reproductive technology that you know?
Ans.
IVF–ET: In vitro fertilization and embryo transfer
GIFT: Gamete intrafallopian transfer
ICSI: Intracytoplasmic sperm injection
TESE: Testicular sperm extraction
MESA: Microsurgical epididymal sperm aspiration
PESA: Percutaneous epididymal sperm aspiration
19E. ONCOLOGY
 
CASE 13: HYSTERECTOMY AND OOPHORECTOMY
Q.134 What are the risks of bilateral oophorectomy during hysterectomy?
Ans. Risks are mainly due to loss of endocrine function of the ovaries. Loss of estradiol function is significant. Moreover, ovarian stroma secretes androgen precursors which are converted to estrogen in adipose tissue (peripheral endocrine organs). Major health hazards following oophorectomy are osteoporosis, fracture and cardiovascular risk. Additional morbidities are due to atrophy of the genital and urinary organs (dyspareunia, dysuria), vasomotor instability (hot flushes) and psychological disturbances (irritability, insomnia, mood swing).
Q.135 What are the benefits of doing bilateral oophorectomy during hysterectomy?
Ans. Bilateral oophorectomy protects the woman against the risks of developing ovarian cancer (100%) and peritoneal cancer (95%). It reduces residual ovarian syndrome (3–5%) and risks of relaparotomy (5–10%) for any benign or malignant ovarian lesions. It also reduces the risk of breast cancer by 50%. It eliminates the problems of chronic pelvic pain and dyspareunia.
Q.136 What is the risk of age (46 years in this case) in developing ovarian cancer?
Ans. Yes, this age has its own independent risk. The current information about the age and the risk of ovarian cancer as follows:
Age < 40 years = infrequent; age 40–45 years = 15–16/100,000; age 60–70 years = 57/100,000 and the median age for ovarian cancer is 63 years.
Q.137 What are the benefits and risks of hysterectomy alone in relation to ovarian cancer and menopause problems?
Ans. The current information in this regard is, a woman who had undergone hysterectomy alone had a long-term reduced risk of epithelial ovarian cancer. However, hysterectomy is associated with earlier onset of menopause (3.7–4.4 years) even if the ovaries are preserved.
Q.138 What is familial ovarian cancer and how common it is?
Ans. Mutations of certain genes (BRCA1 and BRCA2) are associated with higher risk of ovarian and breast cancers. However, of all the epithelial ovarian cancers 1 out of 10 is hereditary. Familial ovarian cancer has an earlier onset of disease.
Q.139 Is there any current guidelines for prophylaxis of oophorectomy during hysterectomy?
Ans. Nothing is specifically recommended till date. It should be based on individual woman's risk and the family history. However, prophylactic oophorectomy during hysterectomy is performed at a median age of 48 years or 5 years before the index case with the family history of ovarian cancer whichever is earlier. Once oophorectomy is done, management of surgical menopause with nonhormonal or hormonal method should be considered.
All these needs informed consent of the individual concerned.
 
CASE 14: POSTMENOPAUSAL BLEEDING
Q.140 What are the common causes of PMB?
Ans. In the developing countries including India genital malignancy particularly carcinoma cervix is the most common cause of PMB. Other causes are:
  • Atrophic endometritis
  • Endometrial hyperplasia
  • Endometrial polyps
  • Dysfunctional uterine bleeding (DUB)
  • Decubitus ulcer
  • Retained (forgotten) pessary or IUCD (Fig. 1.20)
  • Endometrial carcinoma.
Q.141 Who are the high-risk women to develop endometrial carcinoma?
Ans.
  • Unopposed estrogen stimulation
  • Delayed menopause
  • Hypertension
  • Diabetes
  • Overweight (50 pounds overweight increases the risk by 10 times)
  • Nulliparity
  • 20Polycystic ovary syndrome (PCOS)
  • Tamoxifen therapy
  • Family history of endometrial, breast, ovary or colon carcinoma.
zoom view
Fig. 1.20: Rubber ring pessary. This had been removed from a 70-year-old woman. She presented with postmenopausal bleeding. She had been using this for last 20 years. This forgotten pessary caused ulceration and infection in the vagina. Crust had been formed on its surface due to its prolonged stay in the vagina.
Q.142 What is the significance of family history?
Ans. Lynch II syndrome is observed in about 2% of all such cases of hereditary cancers. Such families display high incidence of hereditary nonpolyposis colorectal cancer (HNPCC), adenocarcinoma of ovary, endometrium, stomach, small bowel and the urinary tract. It is associated with mutations in the DNA mismatch repair (MMR) genes (MSH2, MLH1, PMS1 or MSH6).
Q.143 What is the current change in the trends of diagnosis of endometrial cancer?
Ans. Most gynecologists recommend outpatient endometrial biopsy to confirm the diagnosis. Diagnostic accuracy of outpatient endometrial biopsy using pipelle endometrial sampler is 90–98%. Besides pipelle endometrial sampler, Sherman curette, transvaginal ultrasonography, sonohysterography, hysteroscopy and directed biopsy are also used.
Q.144 What is the place of different procedures in the diagnosis of endometrial carcinoma?
Ans. Fractional curettage is a definite diagnostic procedure for endometrial cancer but it is invasive. Role of hysteroscopy in the diagnosis has certain advantages. To date, there is no universal agreement to the cut-off measurement of endometrial thickness to diagnose endometrial cancer. Endometrial thickness less than or equal to mm in a postmenopausal woman is commonly due to atrophy.
  • Hysteroscopy is more accurate in detecting polyps or submucous fibroids. It helps to see the pathologic lesion as to take direct biopsy. It also helps to evaluate the endocervical canal.
  • Saline infusion sonography (SIS) is helpful in differentiating patients with minimal endometrial tissue from patients with thickened endometrium or polyps.
  • Fractional curettage is mandatory if endometrial sampling procedures fail to provide sufficient material for diagnostic evaluation or when the symptoms are persistent in spite of negative endometrial sampling.
  • Considering the benefits most clinics prefer to do outpatient (office) endometrial biopsy as a first diagnostic step. When biopsy result is negative and further evaluation is needed, hysteroscopy and biopsy is done.
Q.145 What are the characteristics of type I and type II endometrial carcinoma?
Ans. Differentiating features of type I and II endometrial carcinoma
Clinical characters
Type I
Type II
Risk factor
Unopposed estrogen
Age
Occurrence
Common (85%)
Less common
Predisposing factor
Hyperestrogenic state
No such
Precursor endometrium
Atypical hyperplasia
May be atrophic
Age
Young (perimenopause)
Older (postmenopause)
Endometrial hyperplasia
Present
Absent
Tissue differentiation
Well
Poor
Myometrial invasion
Minimal
Deep
Histology
Endometrioid
Serous, clear
Molecular characters
Ploidy
Polyploid
Aneuploid
HER2/neu overexpression
No
Yes
P-53
No
Yes
PTEN mutations
Yes
No
Prognosis
Favorable
Not favorable
Q.146 Are all endometrial cancers due to estrogenic stimulation?
Ans. Currently, two pathogenic types of endometrial carcinoma are being observed.
21Type I: The majority (75–85%) belong to this group. Women having a persistent stimulation of endometrium with unopposed estrogen either endogenous or exogenous, run the risk of developing type I endometrial carcinoma.
Type II: Women with type II develop endometrial carcinoma without any estrogenic stimulation. Carcinoma develops not from the background of endometrial hyperplasia but may arise from an atrophic endometrium.
Estrogen dependent tumors develop in relatively younger perimenopausal women as opposed to the estrogen independent tumors that occur in older postmenopausal women. Tumors that are estrogen dependent have favorable prognosis compared to estrogen independent tumors. Based on molecular genetic studies, these two types of tumors have different pathogenetic pathways.
Q.147 What is the current situation of TVS, CT and MRI in the evaluation of endometrial cancer?
Ans. MRI is found to be superior to conventional CT in the assessment of depth of myometrial invasion. Studies have shown diagnostic accuracy of TVS is about the same (68–69%) compared to T2-weighted MRI (68–74%) in evaluating myometrial invasion. However, contrast enhanced MRI is always superior to TVS and is found to be better when compared to CT even.
The accuracy of MRI in staging endometrial carcinoma has been reported to be 85% overall. Gadolinium enhanced images significantly improve accuracy.
There are certain clinical situations where assessment of endometrial carcinoma may be difficult (e.g., obesity) or may not be possible (e.g., contraindications for surgical staging or because of tumor spread). In such situations MRI has got distinct advantages to evaluate the disease.
Suggested Reading
  1. Walker JL, Piedmonte MR, Spirots NM, et. al. Laparoscopy compared with laparotomy for comprehensive surgical staging of uterine cancer: Gynecologic Oncology Group Study LAP2. JClin Oncol. 2009;27:5331–6.
 
CASE 15: CARCINOMA CERVIX—A
zoom view
Fig. 1.21: Carcinoma cervix showing an exophytic growth with ulceration.
Q.148 Clinically, what are the different types of the lesion?
Ans. (i) Exophytic, (ii) Ulcerative, and (iii) Infiltrative.
Q.149 Histopathologically, what are the different types of carcinoma cervix?
Ans. (i) Squamous cell carcinoma (75–80%), (ii) Adenocarcinoma (20–25%), (iii) Adenosquamous carcinoma, (iv) Neuroendocrine tumors, and (v) Others: Lymphomas.
Q.150 What are the different modes of spread in carcinoma cervix?
Ans.
  • Direct extension (parametrium, endocervix, vagina, bladder)
  • Lymphatics (pelvic and para-aortic)
  • Direct implantation
  • Bloodstream (lung, mediastinum, bone, liver).
Q.151 What are the complications that may arise in a case with carcinoma cervix?
Ans.
  • Hemorrhage
  • Pain in the loin due to pyelitis, pyelonephritis
  • Vesicovaginal fistula
  • Rectovaginal fistula
  • Uremia
  • Sepsis.
Q.152 What are the advantages of radiotherapy as a primary modality of therapy?
Ans.
  • Wider applicability in all stages of carcinoma cervix.
  • Survival rate of radiotherapy (85%) is comparable to that of surgery in early stages.
  • Less primary mortality or morbidity.
22Q.153 Mention the different types of hysterectomy that can be done for the management of microinvasive and early invasive carcinoma cervix?
Ans. Classification of extended hysterectomy (ACOG–1974).
Rutledge's classification of extended hysterectomy (1974)
Class
Description
Indication
I
Extrafascial hysterectomy; pubocervical ligament is incised, allowing lateral deflection of the ureter
CIN, early stromal invasion
II
Removal of the medial half of the cardinal and uterosacral ligaments; upper third of the vagina
Microcarcinoma post-irradiation
III
Removal of the entire cardinal and uterosacral ligaments; upper third of the vagina removed
Stage IB and stage IIA tumors
IV
Removal of all periureteral tissue, superior vesical artery and three-fourths of the vagina
Anteriorly occurring central recurrences
V
Removal of portions of the distal ureter and bladder
Central recurrent cancer involving bladder and ureter
Q.154 What are the preventive measures against carcinoma cervix?
Ans. Primary prevention
  • Identifying and preventing the high-risk factors: Preventing human papillomavirus (HPV) infection with HPV vaccine to all school girls (9–15 years).
Other risk factors (cofactors) to avoid are:
  • Early sexual intercourse
  • Early age of first pregnancy
  • Multiple partners
  • Too many births and too frequent births
  • Poor local hygiene.
Secondary prevention:
  • Screening against carcinoma cervix: cytology (liquid-based cytology), HPV testing (hybrid capture II), visual inspection with acetic acid (VIA), colposcopy and biopsy.
  • ‘Down staging screening’ by WHO—intends to diagnose the disease early and to minimize cancer death.
Q.155 What is the role of HPV in the pathogenesis of carcinoma cervix?
Ans. HPV is a DNA virus which is epitheliotropic. High oncogenic risk HPV (types 16, 18, 31, 33, 35, 45, 56) types are responsible in the etiopathogenesis of cancer cervix (Fig. 1.22).
zoom view
Fig. 1.22: Pathogenesis of HPV infection showing normal stratified squamous epithelium on the left and its progression to CIN/CIS and invasive on the right. The grades of CIN (I to III), CIS and the invasive carcinoma are shown.
Over 99.7% of patients with CIN and invasive cancer are found to be positive with HPV-DNA. Infection always starts at the transformation zone (TZ) of the squamocolumnar junction (SCJ). Once the high-risk oncogenic HPV-DNA integration occurs within human (host) genome (infection), there is overexpression of E6 and E7 oncoproteins. These oncoproteins suppress the tumor suppressor genes [P-53 and retinoblastoma (Rb)]. This results in neoplastic transformation of the mitotically active metaplastic epithelium of the ‘transformation zone’ at 23the level of SCJ (See Dutta's Textbook of Gynecology, 8th Edition, p. 269). Normally the tumor suppressor gene P-53 will cause infected cell death by apoptosis and thus halting the viral multiplication. But these oncoproteins cause proteolytic degradation of P-53, resulting in cell immortalization and viral multiplication. Currently, HPV vaccines are available. Vaccines are very effective in preventing infection with HPV. Vaccines (Cervarix, Gardasil) are approved to all school girls (9–15 years) and women (16–26 years). Immunity persists for about 5–7.5 years. Vaccines are type specific and effective only when used prophylactically. Currently nonvalent vaccines are available. Single dose vaccine may be used and is found to be effective (61.6%).
Q.156 What are the advantages of surgery in the management of carcinoma cervix, over radiotherapy?
Ans.
  • Thorough surgicopathological staging during surgery.
  • Accurate prediction of survival rate by para-aortic and pelvic node assessment surgically.
  • Preservation of ovarian function when desired.
  • Retention of more functional and pliable vagina.
  • Transposition of ovaries when needed for consideration of future radiotherapy.
  • Psychological benefit of the woman.
Q.157 What is neoadjuvant chemotherapy and what are the benefits?
Ans. Platinum-based combination chemotherapy is given for three cycles. This is followed by radical hyterectomy and pelvic lymphadenectomy. Neoadjuvant chemotherapy improves the resectibility of bulky (4 cm) disease (stage IB2 and stage IIA).
Q.158 What is concurrent chemoradiation?
Ans. It is the combined therapy with radiation and weekly cisplatin-based combination chemotherapy. This therapy is found helpful as a primary treatment for stage IB, IIA or advanced stage (IIB to IVA) disease.
Q.159 How the radiation dose is calculated in the management of cancer cervix?
Ans. Radiation dose is calculated with respect of radiation received at two arbitary points—point A and point B.
  • Point A is 2 cm cephalic and 2 cm lateral to the external os. It is the point of crossing of the uterine artery and ureter.
  • Point B is 2 cm cephalic and 5 cm lateral at the same plane. It is approximately the site of obturator node.
  • Point A gets about 7000–8000 cGy and point B 2000 cgy from radium (brachytherapy). Cancerolytic dose is approximately 7000–75000 cGy. The rest of the dose at point B is supplemented by external beam radiation.
Q.160 What is the overall 5 years survival rate following therapy in a woman with carcinoma cervix?
Ans.
STAGES
5 years survival rate (%) (International Federation of Gynecology and Obstetrics)
IA1
IA2
98.7
95.9
The overall 5 years survival for all the stages is 1.9%
Important prognostic factors are: tumor volume, lymph node metastasis, parametrial invasion and lymphovasular space invasion
IB1
IB2
88.0
78.8
IIA
IIB
68.8
64.7
IIIA
IIIB
40.4
43.3
IVA
IVB
19.5
15
Q.161 How cervical cancer screening could be organized in a low-resource setting?
Ans. Cytology based cervical cancer screening program (India, Pacific islands) could not be successful, in many countries of the world, as it requires a number of procedures. Alternative cervical cancer screening strategies in low resource settings can have a consistent and significant impact to improve upon the burden of cancer deaths.
Recommendations (ACOG-2015): The following are the acceptable alternatives where cytology based screening is not feasible or practical.
  • HPV testing and follow-up treatment for woman with positive test results.
  • In settings where HPV-DNA is not available, visual inspection with acetic acid followed by treatment with cryotherapy to be done. Cryotherapy should not be done in women where squamocolumnar junction is not entirely visualized. It has been established with a prospective randomized study in rural India that a single life time HPV-DNA screening test can reduce cervical cancer mortality and late stage disease by about 50%.
Q.162 Discuss the sensitivity of different screening procedures to detect CIN2+.
Ans. Sensitivity of screening to detect CIN2+
Test
Sensitivity (%)
1. Conventional pap
55–82 (63)
2. Liquid-based cytology
57–90 (74)
3. VIA
41–79 (60)
4. HPV
94–98 (96)
5. HPV + Pap test
99–100 (99+)
24Q.163 What is the strategy of WHO for the elimination of cervical cancer by 2030?
Ans.
ROAD TO ‘ELIMINATE’ CERVICAL CANCER
THRESHOLD: All countries to reach < 4 cases 100,000 women-years
2030 control targets
90%
of girls fully vaccinated with HPV vaccine by 15 years of age
70%
of women screened with an high precision test at 35 and 45 years of age
90%
of women identified with cervical disease receive treatment and care
30%
reduction in mortality from cervical cancer
SDG 2030: Target 30% reduction in mortality from cervical cancer
The 2030 targets and elimination threshold are subject to revision depending on the outcomes of the modeling and the WHO approval process.
Q.164 What are the aims of palliative treatment? What different palliative treatments can be given to a woman with carcinoma cervix?
Ans. Palliative treatment is aimed to provide comprehensive care for relief of symptoms along with treatment of the cancer, in the advanced stage.
Palliative treatment is given:
  • To control vaginal discharge
  • To control hemorrhage
  • To relief pain.
Discussion continued: Case 16.
Suggested Reading
  1. Clavel C, Masure M, Bory JP, Putaud I, Mangeonjean C, Lorenzato M, et al. Human papillomavirus testing in primary screening for the detection of high-grade cervical lesions: a study of 7932 women. BJC. 2001;89:616.
  2. Cuzick J, Clavel C, Petry KU, et al. Overview of the European and North American studies on HPV testing in primary cervical cancer screening. IJC. 2006;119:1095-101.
  3. Ronco G, Segnan N, Giorgi-Rossi P, et al. Human papillomavirus testing and liquid-based cytology: results at recruitment from the new technologies for cervical cancer randomized controlled trial. JNCL. 2006;98:765-74.
  4. Sangrajrang S, Laowahutanont P, Wongsena M, Muwonge R, Karalak A, et al. Comparative accuracy of Pap smear and HPV screening in Ubon Ratchathani in Thailand. Papillomavirus Res. 2017;3:30-35.
  5. Sankaranarayanan R, Gaffikin L, Jacob M, Sellors J, Robles S. A critical assessment of screening methods for cervical neoplasia. Int J Gynaecol Obstet. 2005;89(Suppl 2):S4-S12.
  6. Sankaranarayan R, Nene BM, Shastri SS, et al. HPV screening for cervical cancer in rural India. N Engl J Med. 2009;360:1385-94.
  7. Srivastava AN, Misra JS, Srivastava S, Das BC, Gupta S. Cervical cancer screening in rural India: Status and current concepts. IJMR. 2018;148(6):687-96.
 
CASE 16: CARCINOMA CERVIX—B
Q.165 What is the stage IIA carcinoma cervix?
Ans. Carcinoma of the cervix extending downwards up to upper two-third of the vagina but not laterally (without parametrial invasion).
It is subdivided into:
IIA1: Clinically visible lesion less than or equal to 4.0 cm in greatest dimension.
IIA2: Clinically visible lesion greater than 4 cm in greatest dimension.
zoom view
Fig. 1.23: Carcinoma cervix—radical hysterectomy done. Tied ends of uterine arteries are seen as they were dissected out and severed from the origin.
Q.166 What does this radical hysterectomy mean?
Ans. Removal of the uterus, cervix, upper vagina, parametrium, pelvic lymph nodes. Para-aortic node sampling is also done.
Q.167 What are the important complications of the operation?
Ans. Anesthetic complications and complications due to surgery: hemorrhage, injury to bladder, bowel, ureters, infection, intestinal obstruction and lately fistula and lymphocyst formation.
25Q.168 What are the consequences if she refuses the operation?
Ans. Progression of the disease and the problems of bleeding, sepsis, metastases, cachexia and renal failure (uremia).
Q.169 What is the long-term outcome following the operation?
Ans. If the nodes are not involved, 5-year survival rate is about 80%. On the other hand 5-year survival rate is about 50% if nodes are involved.
Q.170 Is there any alternative method of treatment?
Ans. Radiotherapy (combined teletherapy and brachytherapy) has got equal survival rate for this stage. Chemoradiation therapy is also effective. (See Dutta's Textbook of Gynecology, 8th Edition, p. 295).
Q.171 What is the place of fertility sparing surgery for carcinoma cervix?
Ans. Fertility-preserving surgery for carcinoma cervix is done in highly selected women (See Ch 39).
Indication: Woman keen to preserve her fertility and has no other factor for infertility.
Case selection criteria
  • Early stage disease (See Dutta's Textbook of Gynecology, 8th Edition, p. 295)
  • Woman is strongly motivated for follow-up
  • Invasive disease has been excluded on thorough examination
  • Tumor is not a highly aggressive histologic subtype (neuroendocrine tumor).
Stage (FIGO) of the disease and the type of operation
Stage (FIGO)
Operation (fertility preservation)
  • Microinvasive carcinoma stage 1A1 (no LVSI)
  • Stage 1A1 with
    • Lymphovascular space involvement (LVSI)
    • Stage 1A2
    • Stage 1A (adenocarcinoma)
    • Stage IB1 (tumor volume ≤ 2 cm)
  • Cervical conization (cold knife cone)
  • Radical trachelectomy with lymphadenectomy (laparoscopic robotic assisted)
  • Radical trachelectomy removes most of the cervix, bilateral parametria and upper vagina
  • Uterus is preserved for child-bearing function. It can be done vaginally or abdominally (laparoscopically or a robotic-assisted laparoscopy)
Q.172 What are the serum tumor markers of carcinoma cervix?
Ans. Serum tumor markers are valuable for predicting the prognosis, the response to treatment and also the risk of recurrence. In carcinoma cervix, the role of serum tumor markers in predicting 5-year survival rate is limited.
The most commonly studied tumor markers are:
  • Squamus cell carcinoma antigen (SCCA)
  • Cancer antigen 125 (CA 125)
  • Cytokeratin fragment 19 (CYFRA 21.1).
  • Raised levels of SCCA, CYFRA 21.1 and CA 125 have been observed in 20–88% of women with cervical cancer. Raised levels are correlated with tumor stage, tumor volume, cervical stromal invasion, lymphovascular space invasion, parametrial involvement and lymph node metastasis.
Q.173 What is the role of radiographic studies in tumor evaluation?
Ans. According to FIGO, radiographical studies such as CT, PET, combined PET/CT and MRI may be used to determine the extent of the disease within the pelvis, lymph nodes, for evaluation of prognosis and risk of recurrence following of initial treatment. However, imaging and pathology can be used, where available. In that case, adding notation of r (imaging) and p (pathology) has to be made.
  • MRI is more accurate to determine tumor diameter, uterine extension and parametrial involvement compared to CT or clinical examination.
  • PET/CT is more valuable to evaluate nodal metastases.
  • Dynamic contrast-enhanced MRI can evaluate tumor vascularity and perfusion. [18F] fluorodeoxyglucose PET: PET/CT can evaluate intratumoral metabolic actively. All these can predict response to therapy. Discussion continued: Case 17.
 
CASE 17: CARCINOMA CERVIX—C
Q.174 What is stage IA carcinoma cervix?
Ans. Carcinoma strictly confined to the cervix and that is in the preclinical stage. It is diagnosed by biopsy only.
Q.175 Is there any subdivisions of the stage?
Ans. Yes, stage IA1: When there is minimal microscopically evident stromal invasion < 3.0 mm in depth.
Stage IA2: Microscopically measured stromal invasion > 3 mm but < 5 mm.
26Q.176 What is the significance of such classification?
Ans. It is entirely related to spread of the disease and hence directly related to the prognosis and survival outcome.
Q.177 What is the approximate rate of lymph node involvement in this stage and 5-year survival rate?
Ans. Pelvic nodes involvement for stage IA1 (< 3 mm) is 0–0.5% and for stage IA2 (3–5 mm) is 5–6%. The risk of para-aortic nodes involvement is 0 and 1%, respectively. 5-year survival rate for stage IA1 is 98.7% and for stage IA2 is 95.9% respectively. (See Dutta's Textbook of Gynecology, 8th Edition, p. 296).
Q.178 What are the treatment options for this woman?
Ans. Either extrafascial hysterectomy (type I) or modified radical hysterectomy (type II) should be optimum for her depending on whether she belongs to stage IA1 or stage IA2.
Q.179 What is meant by type I and type II hysterectomy?
Ans. Type I: Extrafascial hysterectomy; the pubocervical ligament is incised allowing lateral deflection of the ureter.
Type II (modified radical): Here medial half of the Mackenrodt's and uterosacral ligaments along with selective (clinically enlarged and palpable) lymph nodes are removed. Upper third of vagina is also removed.
Q.180 Can she have any surgery preserving her fertility status?
Ans. Yes, such treatment can be provided in exceptional circumstances only. Therapeutic conization is the option that she can have. In that case, she must realize the need of long-term follow-up. She should be followed up with cytology and colposcopy regularly.
Laparoscopic-assisted radical vaginal trachelectomy with pelvic and aortic lymphadenectomy (LARVT) is also recommended currently for early stage (IA2 and IB1) disease. This is done only for a strongly motivated woman who wants to preserve her fertility (See Dutta's Textbook of Gynecology, 8th Edition, p. 295).
In either of the above treatment options, the surgical margin must be free of disease.
Q.181 What is radical trachelectomy?
Ans. It involves removing the cervix, parametria and cuff of vagina. Body of the uterus is preserved for fertility. This procedure is combined with either extraperitoneal or laparoscopic pelvic lymphadenectomy.
Q.182 What is concurrent chemoradiation?
Ans. Cisplatin-based concurrent chemoradiation is currently recommended as a treatment option of carcinoma cervix. It acts as a radiosensitizer and reduces disease progression. The appropriate cases for this treatment are: (i) Early stage (stages IA2, IB, IIA) disease after radical hysterectomy, (ii) Locally advanced (stages IIB to IVA) disease as a primary therapy.
 
CASE 18: CARCINOMA ENDOMETRIUM
Q.183 What are the important etiological factors for endometrial carcinoma?
Ans.
  • Unapposed estrogen stimulation
  • Age: 50–60 years
  • Parity—nulliparity
  • Late menopause
  • Corpus cancer syndrome (obesity, hypertension and diabetes)
  • Overweight (21–50 pounds overweight increases the risk 3 times)
  • Tamoxifen therapy
  • Positive family history
  • Endometrial hyperplasia.
zoom view
Fig. 1.24: Following surgery, uterus of the same woman is cut open to show a diffused and ulcerated growth at the fundus. The growth is seen to invade the myometrium. Histology confirmed adenocarcinoma
Q.184 What is the common histological type of endometrial carcinoma?
Ans. Adenocarcinoma.
Q.185 What are the high-risk factors for endometrial carcinoma?
27Ans.
  • Persistent and unopposed estrogen stimulation of endometrium
  • Corpus cancer syndrome—obesity, hypertension and diabetes
  • Endometrial hyperplasia
  • Family history of colon, ovarian or breast cancer.
Q.186 How can be endometrial carcinoma diagnosed?
Ans.
  • History and clinical examination: A case of postmenopausal bleeding is considered due to endometrial carcinoma unless proved otherwise.
  • Endometrial biopsy (pipelle) diagnostic accuracy is 90–98%.
  • Papanicolaou smear (unreliable) as diagnosis accuracy is only 30%.
  • Ultrasonography (TVS) including color Doppler, endometrial thickness > 4 mm with increased vascularity is suggestive.
  • Endometrial thickness ≥ 13 mm is strongly suggestive.
  • Hysteroscopy helps in detecting polyps or submucous myomas.
  • Fractional curettage is the definitive method of diagnosis in case where adequate evaluation is not possible with pipelle biopsy specimen or where bleeding recurs after a negative endometrial biopsy.
  • CT and MRI, both can detect myometrial invasion, lymph node involvement and also the spread of the disease. MRI is superior to CT (Figs. 1.25A and B).
zoom view
Figs. 1.25A and B: (A) Magnetic resonance image (MRI) in the sagittal plane of a 55-year-old woman with endometrial carcinoma. Sagittal T1-weighted image shows endometrial tumor with invasion to the myometrium, more than 50%. There is extension of the cancer to the cervix (see arrows); (B) Magnetic resonance image (MRI) in the axial view of the same woman with endometrial carcinoma. Invasion to the myometrium, more than 50% is seen.Courtesy: Department of Obstetrics and Gynecology and Department of Radiology, Assam Medical College, Dibrugarh, Assam, India.
Q.187 Describe orderly the steps of fractional curettage.
Ans.
  • Endocervical curettage
  • Dilatation of internal os
  • Introduction of a polyp forceps to remove any polyp
  • Uterine curettage (1) Uterine fundus, and (2) body of the uterus
  • Specimens, so obtained are sent for histology separately.
Q.188 Mention the important surgical procedures for the management of a case of endometrial carcinoma.
Ans. In stage I: Extrafascial hysterectomy, bilateral salpingo-oophorectomy.
Procedures
  • Incision—longitudinal
  • Peritoneal washings for cytology
  • Thorough exploration of pelvic and abdominal organs including the pelvic and para-aortic lymph nodes
  • Extrafascial hysterectomy (TAH and BSO) is done
  • Uterus is cut open in the theater for tumor evaluation (gross examination or frozen section)
  • Lymph node sampling—pelvic and para-aortic nodes are done when there is myometrial invasion (See Ch 33 and 34).
 
CASE 19: CARCINOMA OF THE OVARY
Q.189 What are the reasons for failure to improve the outcome of ovarian cancer?
Ans.
  • No high-risk factor is known
  • Deep-seated organs—diagnosis is often late
  • No effective preventive measures
  • No preinvasive stage of the disease
  • No effective screening procedure for early detection
  • Spread of the disease is unrelated to the size of the mass and or symptoms of the woman
  • 28Tumor cells move freely within the peritoneal cavity
  • Limitations of cytoreductive surgery, chemotherapy and radiation therapy.
zoom view
Figs. 1.26A and B: (A) Gross photograph of a surgical specimen showing the uterus with a huge ovarian tumor. The other tube and the ovary are seen. Stretched fallopian tube over the tumor is seen; (B) The cut section of the same ovarian tumor to show the solid areas (see arrows). Histology revealed serous cyst adenocarcinoma.
Q.190 What is the place of screening for ovarian cancer?
Ans. Unfortunately, till date no specific method of screening for early detection of epithelial ovarian cancer is available.
  • Clinical methods—bimanual pelvic examination is neither specific nor helpful.
  • Tumor markers CA-125 and HE4 are being done. These are useful but nonspecific (See Ch 17).
  • Ultrasound imaging TVS color Doppler imaging has been used to differentiate a benign from a malignant mass. However, it is not specific also.
  • Risk of malignancy index (RMI) is calculated by U × M × CA 125. The risk of cancer is 75% when the RMI value is greater than 250. When RMI is less than 25 the woman is in the low-risk group. RMI: 25–250 is in the moderate-risk category.
  • Risk of ovarian malignancy algorithm (ROMA): ROMA is a quantitative test using CA-125, HE4 concentration and menopausal status to calculate the risk of ovarian cancer (see below).
  • No screening procedure is effective for early detection of ovarian cancer.
 
Tumor Marker (HE4)
Human epididymis 4 (HE4) protein is derived from the distal epithelium of the epididymis. It is essential for sperm maturation as it is a protease inhibitor.
Levels of serum HE4 is found high in women with serous epithelial ovarian cancer. Unlike serum CA-125, serum levels of HE4 are less affected by other benign pelvic conditions like endometriosis. The sensitivity and specificity of detecting ovarian malignancy with HE4 is high in premenopausal women. This is especially beneficial for early stage ovarian cancer.
The use in combination of serum CA-125 and HE4 was found superior when compared with any other marker alone.
Risk of ovarian malignancy algorithm (ROMA): ROMA calculation is based on the combined results of the CA-125 and the HE4. This algorithm classifies women either as low-risk or high-risk for malignant disease. A cut-off value of 2.27 representing a high-risk of malignancy. Overall, it has a sensitivity of 89% and a specificity of 75%. Both the tumor markers are related to tumor stage and histological type of serous epithelial ovarian cancer.
Q.191 What are the other tumor markers?
Ans. CEA, CDX2, CA 72-4, CA19-9, Alpha FP, LDH and beta-hCG. There is not enough evidence to do the panels of multiple markers as there is no added advantage. All these markers have low sensitivity and wide variation in specificity. The routine use of any of these markers is not recommended (RCOG-2016).
Q.192 What are the epidemiology and risk factors for epithelial ovarian cancer?
Ans.
  • Women of North America and most of the industrialized countries of Europe have high incidence.
  • Pregnancy, breastfeeding reduce the risk.
  • Use of combined oral contraceptives are associated with reduced risk.
  • Pregnancy in a women after the age of 35 years is more protective against ovarian cancer.
  • Use of long-acting progestin only contraceptive (DMPA, Provera) has protective effects similar to COCs.
  • 29Surgery: Tubal ligation (interruption) and hysterectomy can reduce the risk of ovarian cancer.
  • Prophylactic/risk reducing salpingectomy in high-risk women is protective (See Ch 53).
Q.193 What is the place of imaging studies in the management of ovarian cancer?
Ans. Computed tomography (CT)/positron emission tomography (PET) scan has been integrated for the diagnosis of ovarian cancer and evaluation of disease recurrence. PET scan has very high sensitivity but low specificity. There are false positive results due to increased FDG uptake in benign but metabolically active tissues and with inflammatory changes. Combined CT/PET scan had high sensitivity of 100% and specificity of 92%. It can detect recurrent disease which is superior to CA 125 or CT/MRI scans used alone.
Magnetic resonance imaging (MRI) is a nonradioactive imaging modality. It has excellent soft tissue contrast resolution. MRI is better than CT or ultrasonography in the diagnosis of small peritoneal metastases. CT imaging is better in identifying omental metastasis in ovarian cancer.
Q.194 What is the place of lymph node dissection in the managment of ovarian cancer?
Ans. Lymph node dissection in early ovarian cancer: Both pelvic and para-aortic lymph nodes should be removed. Lymph node dissection provides important prognostic and staging information for patients with suspected early stage ovarian cancer. This is helpful for the decision of about adjuvant chemotherapy. However, there is no convincing evidence that lymphadenectomy has got any therapeutic survival benefit in advanced ovarian cancer. Enlarged nodes may be removed as a part of debulking procedure.
Q.195 What is the place of neoadjuvant chemotherapy?
Ans. Neoadjuvant chemotherapy: Women with FIGO IIIC or IV ovarian cancers were treated with neoadjuvant platinum-based chemotherapy followed by primary debulking surgery. Subsequently, they were treated with platinum-based chemotherapy. The blood loss during surgery was less. The largest residual tumor could be reduced to less than 1 cm. The operative time was also shorter. For patients with huge tumor burden, ascites and several comorbidities, neoadjuvant chemotherapy is a very reasonable choice. Surgical debulking procedure is the single most important component in the management of ovarian cancer.
Suggested Reading
  1. Moore RG, Brown AK, Miller MC, et al. The use of multiple novel tumor biomarkers for the detection of ovarian carcinoma in patients with a pelvic mass. Gynecol Oncol. 2008;108(2):402-8.
  2. Rafael Malina, Jose M, Jose Auge, et al. HE4 a novel tumor marker for ovarian cancer: Comparison with CA125 and ROMA algorithm in patients with gynecological diseases. International Society of Oncology and Biomarkers 2011.
 
CASE 20: INHERITED CANCERS AND THE MANAGEMENT
Q.196 Ms RT is keen to know what is inherited or familial cancers. How it could be prevented?
Ans. Familial or inherited cancers are well known. They are more common in relation to ovarian, endometrial, breast and colon cancers. Unfortunately, as regard the diagnosis and prevention, no such specific guidelines are there as yet.
Q.197 What are the diagnostic criteria for familial cancers?
Ans. Till date no such specific guidelines are there. Amsterdam criteria (1999) has considered the following:
  • At least three relations with breast, ovarian cancer and hereditary nonpolyposis colorectal cancer must be there.
  • One affected person is a first-degree relative of the other two.
  • At least two successive generations are affected.
  • At least one person was diagnosed before the age of 50 years.
  • Familial adenomatous polyposis had been excluded.
  • Tumors have been verified by histopathological examination.
Q.198 How do BRCA-1, BRCA-2 and mismatch repair genes protect us?
Ans. BRCA-1 and BRCA-2 are the tumor suppressor genes. Others: MLH-1, MSH-2 and MSH-6 are the mismatch repair genes. These genes repair the single base pair 30mismatches that occur during DNA replication. Mutations in these genes cause cell immortalization, neoplastic proliferation and ultimately cancers.
Q.199 How familial cancers could be prevented?
Ans. At the moment there are no such guidelines. But following measures could be adopted:
  • Family history must be enquired and recorded in all positive cases.
  • Clinical geneticist must be consulted and family tree should be drawn in a positive case (Amsterdam criteria).
  • Molecular screening for detection of gene mutation is not possible at the moment. This is very expensive, time consuming and also labor intensive.
  • Screening procedure may be initiated wherever available (e.g., breast cancer, colon cancer).
  • Combined oral contraceptives are chemopreventive against ovarian cancer and endometrial cancer. Tamoxifen is chemopreventive against breast cancer.
  • Place of prophylactic surgery may be considered. Prophylactic salpingo-oophorectomy/salpingectomy may be done especially while doing hysterectomy for some other reason. This is an alternative in the high-risk group of women for ovarian cancer.
  • Human fertilization and embryology authority (HFEA), UK, has recommended preimplantation genetic diagnosis in cases with familial cancers. Affected blastocysts are removed from the transfer in ART. This may be one way to eliminate the risk of inherited cancers.
  • As a prophylaxis to endometrial cancer, LNG-IUS is currently being used. Long-term reports are awaited.
  • Resequencing chips are being tried, once available molecular diagnosis of gene mutation would be helpful.
Q.200 Who are the women that may be considered for molecular screening?
Ans. At the moment there is no such established criteria. However, women fulfilling the following criteria are the high-risk group for consideration:
  • Positive family history following Amsterdam criteria.
  • Risk scoring more than 10 (Manchester scoring system).
  • Tumors positive for microsatellite instability by immunohistochemistry method.
Q.201 What is the significance of family history?
Ans. This is often observed in early age onset (around 45 years).
Lifetime risk of endometrial cancer in women with Lynch II syndrome is 32–60% and that of ovarian cancer is 10–12%.